Sie sind auf Seite 1von 598
Baglics of Electrical Engineering rn?) SANJEEV SHARMA Prublished tye LK. Imtermationst Publishing House Put. Lie $25, Green Park Extension Uphaar Cinema Market New Dethi-110 016 (India) Esmuil: ik inevsaLset Branch Offices A-6, Royal Indusicial Estate, Naigaum Cross Road Waal, Mumbainsi0 O91 (Inia) E-mail: _snuesbalgessal.act Get “Embassy Centre”, 11 Crescoat Road Kumara Park East, Baagalore-S60 001 ¢india E-mail: ik_bangi@vsal.net ISBN 978-81-89866 9 HT LK, International Poblivhing Howse Pui, Lad. Al rights reserved. No pant of this haok may be repraduced o¢ used in anv form, electronic or mechanical, including photocep¥ing, recording. or bY an¥ information storage and retrieval s¥stem, without writtea permission froa the publisher: Published by Krishan Makijani for LK. locersational Publishing Hlouse Pur. Lid.. $25, Green Paik Extension, Upbaar Cingma Market, New Delht-110 096, Printed by Rekha Printers Pos Lid, Obbla Industrial Area, Phuse Il, New Dethi-110 020, Contents Preface vid 1._Alternating Current Fundamentals and Cireuit 1 Ll Alternating Quantity i 1.2 Alternating Yoltige 1 4 Jaye fi 4 1.4 Advantages of Sing Wave 5 LG Periodic Time 1.7 Frequeney s Phase Differe A 19 Phaser Notaiing L10 Measurements of AC Magnitude 8 LI Effective, Virtual or RooteMcan-Square (RIS) Value of Altemating Quantity LI? Average Value 1g 3 dl 2 LIS Operator j 21 116 Circuit with Pure Resistance Only 2 117 Circuit with Pure Inductance Only 23 LIS Cireuit with Pure Capacitance Only 25 soe Condents 126 Series Resonance or Voltage Resonance 40 L27 Band Width | 1.28 Quality Factor and Selectivity 43 1.20 Parallel Resonance ag 1.30 Derivation for Resonance Frequency As 131 Tinpedance at Resonance a7 1.32 Current Magnification AT L33_ Selectivity. and Band Width. ft 134 Current ot Resonance 49 Solved Example 50 Exomige 24 Similarity of Magnetic & Electric Cireuits 128 2.5 Dissimilarity between Magnetic & Electric Circuits 126 28 Reluetances in Parallel 29 29 Blectric CK'T Corresponds to Magnetic Circuit 131 (Series and Parallel) Exemige et 3. D.C. Network Analysis 145 3d Antroduetiog ns 3.2 Charge 145 Li Electric Curent as 34 Amperes 145 BS Woltaxe 146 3.6 Vollaze Source 146 LLL Voltage Division Rule 152 ‘L12 Current Division Rule —__1i2 Urheberrechilich geschiitztes Material Contents xxvii 3.14 Maxwell's Mesh or Loop Method 154 3.18 Method for Writing the Mesh Equation in Matrix Form. 156 3 1 158 ‘217 Choive of Method Mesh or Nola! 4. LIS Wheat Stone Bridge. 161 3.19 Delta-Star Transformation mn : ° “ Spee bet Eeuslosaatign is Thewnin’ Th 3.22 Steps to Follow forthe Thevnin's Theorem 164 393 Norton's Theowm 65 3.24 Steps to Follow for Norton's theorem 16s 3.25 Maximum Power Transies The 3.27 Superposition Theor 170 2 for iz. Circuil 71 je nse Inst and M 66, 4.1_Differcnt Types of Measuring Instruments 266 3 Absolute 267 4.4 Secondary Instruments 267 48 Effects Used in Measuring Instruments, 268 4.6 Working of Indicating Instruments 2 47 Detect ft 2 4.8 Controlling Torguc 269 4.9_Damn ni we 272 4.10 Moving Irom Instruments 274 4.11 Moving Coil Permanent Magnet Instrument 278 4.12 Dynamometer Type Ammeter and Voltmeter 281 4.13 Dynamometer Type Wattmeter 283 4 Induction Watimeter 286 5 ns is Wattmeter in the Circuit 116 Hot Wire Instrumen: 280 4.17 Enorey Meters. 290 Urheberrechilich geschiitztes Material Contents $3 Ideal Transformer: 3 $4 Working Principle of Transformer (I — 312 43 Phasor Diagram of | — 6 Transformer 313 5.6 Phasor Diagrams on Load 318 5.7 Equivalent Circuit of 5.8 Los omer (1 - 0) 0 nsformei 5.10 Voltage Regulation 333 ‘SLL Testing of a 1-6 Transformer 337 13. Showt Cireuit (SC) Test 339 3.14 Sumpner’s Test 2 6. Polyphase Circuit 396 6.1 Pol r 396 6.2 Generation of Three-phase Voltages, 396 6.3 Phase Sequence 398 64 Advantages of Three-phase Systems 398 as panes: 3 6.6 Voliage and Current Relations in s+/Star) System 400 67 Power in Star System 403 4.8 Topographic Vector Diagre 40: 6.9 Voltage and Current Relation in A-system, 40s 35 106 Urheberrechilich geschiitztes Material 6.14 Measurement of Power in 3.phase Cireuits Alt Exemige Slectromes LE: c ‘D1 Untrodwetion 0 1.2 Principle of Energy Conversion 434 23 Int ne 2.4 General Consiruction of Rotating Machine 436 16 Types yaure Winding: 2 27 Worki inciple of DC Machit 4 19 Torque in a DE Motor 4a? Sipe] rpc ‘LU Types of DC Machines, 450 ‘L12 Power Stages in DC Generators 453 13 Power St > Maik 1S. 14 Losses and Efficiency of DC Machines 454 LAS ‘Types of Torque in DC Mators 455 Lec istics of DCG. 5 18 Operating Charneteristies of a DC Motor 463 1.19 Applications of DC Motors 470 Significance of Back EMF 470 Speed Cay 2 Moto 7 of DC Motors 9 . Sh . 97 Namertcal problems 537, 3. Synchronous Machine SM SuUntroducting 4.3 Synehronous Generator or Aliernator. 543 $4 EMF Equation 344 Urheberrechilich geschiitztes Material Contents $8.3 Frequeney of Induced EME $47 $8.6 Advantages of Field Winding on Rotor and 549 Stationary Armature Winding. S¥nehronous Motor 54! “ue Princinheavehoonam = Self-starting Motor) 8.9 How tw Get Continvous Unidirectional Torque? 551 @_ Making S¥Ynehronous Motor Sclf-s it 55, ALLL Starling of Syn-motors 553 8.12 Equivalent CK 553 B13 V-Corve 558 8.14 Rotating Magnetic Field 3586 8.15 Characteristic Beatures, Advantages and Disadvantages $62. 8.16 Applications 563 9.3 Comparison of Squirrel Cage and Slip Ring Induction Motor 568 9.4 Principle of Operation 568, 2S Induct ‘Task 1 9.6 Slip Speed 570 cy of Rotor EMF/Current. 57) 9.8 Power Flow Diagram. S71 2 2 EC i x 9.10 Torque-slip Characteristics 514 9.11 Starting of 3-f VM (Need af Starters) 519 212 Methods 01ST 9.13 Application of 3-f Induction Motor 585 9.14 Comparison of Induction Motor and SYnchronous Motor___586 Solved Examples 587 Urheberrechilich geschiitztes Material Catents i 10.3 Working Principle 610 10.4 Double-field Revolving Theory oul M.S Method of Starting of Single-Phase Induction Motor 613 10.6 Split Phase Method or Split Phase Motors 613 Appendix 1 Solved Questions Papers of Previous Y¢ Appendix 2 Objective Questions for Practical Quix 675 Appendix 3 Electromagnetism: S88 Mndey Urheberrechilich geschiitztes Material Alternating Current Fundamentals and Circuit 4.4 ALTERNATING QUANTITY An alternating quamity is that which acts in alternate directions and whose magnitude undergoes a definite ¢yele of changes in definite intervals of time When a simple loop revolves in a magnetic field, an alternating emfis induced in the loop. Ifthe loop revolves with an uniform angular velocity the induced alternating emf is sinusoidal in nature. The alternating quantity may have various ather wave farms like triangular, semicircular, stepped. distaried, ete as shown in Fig. 1M), (6), (¢) and (a), respectively. The graph repeats ster regular intervals. One complete sct of positive and negative valucs of an allernating quantity is called a eyele, The important alternating quantities, /() that will be discussed in the chapter are cursent and voltage 1.2. ALTERNATING VOLTAGE Abemating voltage may be generated by (9) By rotating 2 coil in a stationary magnetic fiel (b) By rotating a magnetic field within a stationary coil, The value of the voltage (i) The number of turns in the coils, ii} The strength of the field, {iii} The speed at whieh the coil or magnetic field rotates. (a) Maxi position (perpendicular) to the dire {b) When the plune of a coil is horizen ted in each case depends on: uum flux links with the coil when its plane is in vertical m1 OF flux between the pales. ial no flux links with the coil Basics af Electrical E axis {Positional cod at enyinetant Fig. 1.2 (e) Ifat any instant the flux linked 9 is given by = $0058 where 6,,= Max lux which can link with the coil, f= Time token by coil to mave through an angle @ from the vertical position and iv is the angular velocity, then 6 = wr and = 0,, eos wi Aernating Cornent Fiendasentals avd Civewid 3 ao dt WN Ogun Si WT Instantaneous emt e = + Number af cveles‘sce is called the frequency of the alternating quantity. + Time taken by an alternating quantity to complete the cYele is called its lime period, rau = ‘ i tor a=wt A meycle = WT . Fig. 13. 1.3 AC WAVEFORMS When an alternator produces AC voltage, the voltage switches polarity over ime, but does so in 2 ¥ “wave” traced by this valtage of alternating polarity from an altemator takes ona distinet shape, known as a sine wave: '¥ particular manner, When graphed over time, the Tine Flo.1.4 Grashof AC voltage over line (the sine wave) In the volt plot from an eleetromechanical alternator, the change from one polarity to the other is 8 smooth ane, the voltage level changes most rapidly at the zeco (“crossover”) point and most slowly at its peak. If we were {0 graph the trigonometric function of “sine” over a horizontal range of 0 (0 360 degrees, we would find the exact same pattern: 4 Basics af Electrical E Angle in degrees Sine (anole) o : 00000 — sere 15 o.25e8 30 . 0.5000 45 - 7071 60 0.8860 15 : 0.9650 90 11,0000 — positive peak 105 0.9659, 120 0.8660, 135 o7ort 450 0.5000 185 : 0.2588 180 - 0.0000, — zero 195 0.2568 210 0.5000 225 -07071 240 0.8660 255 9.9669 270 : ~ 1.0000 235 : ~0.9659 300 ~0.8660 M5 - 0.7071 330 -0.5000, us 0.2588 360 0.0000, — zero The reason why at eleetiomechanieal alternator produces output sine~ @ AC is due to the physies of its operation. The voltage produced by the stationary coils bY the motion of the rotating magnet is proportional to the rate at which the magnetie flux is changing perpendicular to the coils (Paraday’s Law of Electromagnetic Induction). That rate is greatest when the magnet poles are closest to the cails, and least when the magnet poles are furthest from the coils. Mathematically, the 1a i e due to a rotating magnet follows that of a sine function, so the voltage produced by the coils follows that same function If we were to follow the changing voltage produced by 2 coil in an alternator from any point on the sine Wave graph to that point when the wave shape bevins to repeat itself, we would have marked exactly one evele of that wave. This is most easil¥ shown by spanning the distance between identical peaks, but may be measured between any corresponding points on the graph, The degree marks on the horizontal axis of the graph represent the damain of the trigonometric sine fianction and also the angular position of our simple two- pole alternator shaft as it rotates, “ Aernating Cornent Fiendasentals avd Civewid 5 1.4 ADVANTAGES OF SINE WAVE |. Any periodic non-sinusoidal wave can be expressed as the sum af a number of sine wave of different frequencies Sine wave ean be expressed in a simple mathematical form. 3. The resultant of two or more quantities var¥ing sinusoidally at the same frequency is another sinusoidal quantity of same frequency. 4. Rate of change of any sinusoidal quantity is also sinusoidal. 1.5 CYCLE A cycle may be defined as one complete set of positive and negative values of an alternating quanlily repeating at equal intervals. Each complete spread over 360° electrical as shown in Fig. 1.5 Fig. 4.5 1.6 PERIODIC TIME The time taken by an alternating quantity in seconds to trace one complete cele is called periodic time or time-period, It is usuall¥ denoted by s¥mbol 7 4.7 FREQUENCY The number af e¥cles per second is called frequency and is denoted by symbol. 1 Thus, == ee or, T=— f Lf the angular velocity ws is expressed in radians per second, then an r any w 6 Basics af Electrical 1,8 PHASE DIFFERENCE Let OP and O@ be the two vectors (more . preferred to be called phasors) representing 2) two alternating quantities of the same frequency at an¥ instant. The angle between them is called the phase angle. The diteetion of rotation in counter elock- wise dirction is usually taken as positive. If OG and OF represent voltage and current vectors, then © = OO sin wt and, = OP sin (wr— 0) where, is called the phase difference. In above phasor 0 is said 10 lead the phasor OP, The ‘phase’ of an AC reference axis or refere Phose angle as the angle of lead or lag with respect t reference axis or Wwe may be defined as its position with respect toa we wave AB => Phase shit = «degrees A's anead of B because A attain i maximum of minimum belore 5 Vs aleade a NB lage, wi2 Fig. 7 with respeet to another wave. + fis degree abead of 8. + A attains its maxima or degrees before Bor “LY second degrees or x a nd s sec before B before Aernating Cornent Fiendasentals avd Civewid 7 1.9 PHASOR NOTATION Sinusoidal quantities can be represented by a function. fity= Vie Fae *K This function has constant magnitude ¥,, and as wf moves through 0 ot 27 radians OA=¥,, 0080, OB = ¥,, sin @ OB O€ = (04) + 108) 8 = tan 1 — N+ HOB) OA b cos O +f sin B tangular form Oo OC) =F 447 where O = tan | yix 204 +j 08 acs Jeary Fe My £0, Fe Me, 20 Then Fs = Viyhy, 28, +85 20s (8, * 83)+j sin (B, + O3)] [eos (0, ~ 84) + sin (@, ~ 049] 8 Basics af Electrical Phasor disgram: Let a=W _ Zot (means 1; leads, # bY angle a”) Fig. 1 1.10 MEASUREMENTS OF AC MAGNITUDE So far we know that AC voltage alternates in polarity and AC current alternates in direction. We also know that AC can alter: of different ways. and by tracing the allernation over lime weean plot it as a “waveform”, We ean measure the rate of alternation by measuring the time it takes for a wave to evalve before it repeats itself (the “period”, and express this as e¥cles per unit time, of “frequency”, In music, frequency is the same as pitch, which is the property distinguishing one note from another. essenli However. we encounter a measurement problem if we tr to express how large or small an AC quantity is, With DC, where quamities of voltage and current are generally stable, we have little trouble expressing how much vollage or current we have in any patt of a cireuit, But how do You grant a single measurement of magnitude to something that és constantly changing? One way to express the intensity, or magnitude (also called the amplitude) ofan AC quantity is to measure its peak height on a waveform graph, This is known as the peak or crest value of an AC waveform: Pea “ / Time Fig. 1.10 Another way isto measure the total height between opposite peaks. This is known as the peak-to-peak (P-P) value of an AC waveform. Aernating Cornent Fiendasentals avd Civewid 9 Untvrtunately, either one of these expressions of waveform amplitude ean be misleading when comparing two different types of waves. For example, a square wave peaking at 10 volts is obviously a greater amount of voltage for a greater amount of time than a triangle wave peaking at 10 volts. The effects of these two AC voltages powering a load would be quite different, Peaksiospeak / » , \ Tee Fig. ttt Time ‘ore neat ., Less heat wove 32% enor 4 enatay (pron) Fo" esssipated dissipated (Samo load resistance) Fig. 1.92 One way of expressing the amplitude of different wavesshapes in a more equivalent fashion is to: mathematically average the values of all the points on the graph of a waveform to a sifigle, aggregate number. This amplitude measure is known as the avernge value of the waveform. If we average all the points on the waveform algebraically (that is, to consider their ign, either positive or True average value of al palais (considering thar signe} is zero acgative), the average Fig, 4.12 10 Basics of Electrical E value for most waveforms is technically zero, because all the positive points cancel all the negative points over a full e¥cle This, of course, will he true for any waveform having equalarea portions above and below the “zera” line of plot. Hawever, as a practical measure of a waveform’s aggregate value, “average” is usually defined as the mathemat mean of all the points’ absolute values over a cycle, In other words, we caleulate the practical average value of the waveform by considering.all potnts on the wave as positive quantities as if the waveform looked like this: Practical average of point alues assurmed (0 be posinvee Fig. 1.14 Polarity-insensitive mechanical meter movements (meters designed to respond equally 10 the positive and negative half-eyeles of un alternating voltage or current) register in proportion to the waveform’s (practical) average value, because the inertia of the pointer against the tensioa of the spring naturally averages the force produced by the varying voltage/eurrent values over time. Conversely polarity-sensitive meter movements vibrate usclessly if exposed (AC voliage of current, their needles oscillating rapidly about the ating the true (algebraic) average valu of zero for a s¥mmetrical waveform. When the “average” value of a waveform is referenced in this text, it will be assumed that the “practical” definition of average is intended unless otherwise specified zero mark, Another method of derivin based on the waveform’s ability to do useful work whea applied to a load resistance. Unfortunately, an AC measurement based an work performed by a waveform js not the same as that waveform’s average value, beeause the power dissipated by a given load (work performed per unit tine) is not directly proportional to the magnitude of either the voltage or current impressed upon it. Rether, power is proportional to the sare of the voltage or current applied toa resistance (P= F/R, and P= PR), Although the mathematics of such an amplitude measurement might not be straightforward, the utility of it, is. gregate value for waveforin amplitude is Current would produce the same amount of heat ene through an equal resistance: EY dissipation Aernating Cornent Fiendasentals avd Civewid " + SARMS sow 10W 2% power RMS: anes dissipated + SARMS & ~ . Equal power dsseated through +5A 7 SO Gul eislance acs sow a 18v 2a Se bponr Siesipates 5A Fig. 4.48 In the tWo circuits above, we have the same amount of load resistance (2 22) dissipating the sume amount of power in th powered by AC and the other hy DC. Beeause the AC voltage source pictured above is equivalent (in terms of power delivered 10 @ load) to a 10 vol DC battery, we would call thisa “10 volt" AC source. Mare specifically, we would denote its voltage value es being 10 volls RMS. The qualifier “RMS” stands for Root Mean Square, the algorithm used to obtain the DC equivalent value from point on a graph (essentially, the procedure consists of squaring all the positive and negative points on a waveform graph, averaging those squared values, then taking the square root of the average to obi Sometimes the alternative terms equivalent or DC equivalent are used instead of “RMS”, but the quantity and principle are both the same. form of heat (50 watts), one the final answer), RMS amplitude meastirement is the best Ww quantities, or other AC quantities of differing wavefern shapes, wh with measurements of electric power. For other considerations, peak or pea to-peek measurements may be the best to employ. For instance, when Uciermining the proper size of wire (ampacitY) to conduct electric power from # source ta load, RMS current measurement is the best to use, because the principal concern with current is overheating of the wire, which isa function of power dissipation caused by current through the resistance of the wire, However, when rating insulators for service in high-voliage AC applications, peak voliage measurements are the mast appropriate, because the principal concern here is insulate “flashaver” caused by brief spikes of voltage, inrespective of time. Peak and peak-to-peak measurements are best performed with an ascillascope, which can capture the crests of the waveform with a high degree of accuracy duc to the fast action of the cathode-raytube in response to 12 Basics af Electrical E changes in voltage, For RMS measurements, analog meter movements (D° Arsonval, Weston, iron vane, electrodynamometer) will werk so long as they have been calibrated in RMS figures. Because the mechanical inertia and dampening effects of an clectromechanical meter mavernent makes the deflection of the needle naturally proportional 10 the average value of the AC, not the true RMS value, analog meters must be specifically calibrated (or mi calibrated depending on how You look at it) to indicate voltage or cureent in RMS units. The accuracy of this calibration depends on an assumed waveshaps, usual ine wave, Electronic meters specifically designed for RMS measurement are best for the task. Some instrument manufacturers have designed ingenious methods for determining the RMS value of any waveform. One such manufacturer produces “Truc-RMS” meters with atiny resistive heating element powered bY a voltage proportianal 10 that being measured. The heating effect of that resistance clement is measured thermally to give a truc RMS value with no mathematical calculations whatsoever, just the laws of physics in action in fulfilment of the definition of RMS. The accuracy of this type of RMS measurement is independent of waveshape. For “pure” waveforms, simple conversion coefficients exist for equating peak, peak-to-peak, average (practical, notalgebraic), and RMS measurements to one another RMS =0-707 (Pack) AVG = 0.687 (Peak) . FMS = 0-577 (Peak) “ . o RYG=0.8xPak) (Peat) Fig. 1.18 Aernating Cornent Fiendasentals avd Civewid 13 In sedition 1 RMS average, peak (crest) and peak-to-peak measures of an AC waveform, there ace ratios expressing the proportionality between sone of these fuadamental measurements. The crest factor of an AC waveform, for instance, is the ratio of its peak (crest) value divided hy its RMS value. The form factor of an AC waveform is the ratio of its RMS value divided by average value. Square-shaped waveforms always have crest and form factors equal to 1, since the peak is the same as the RMS and average values Sinusoidal wavefonms have crest factors of 1.414 (the square mot of 2) and form factors of L.1l, Triangle- and sawtooth-shaped waveforms have crest values of 1,732 (the square root of 3) and form factors of 2 Bear in mind that the conversion constant shown here for peak, RMS and age amplitudes of sine waves, square waves, and triangle waves hold tac only far pure forms of these waveshapes. The RMS and average values of distorted waveshapes are not related by same ratios. Rus 2277 po IGE TTT \ PP =2 (Peak) Fig 17 This isa.very important concept to understand when using an analog meter movement to measure AC voltage or current, An analog movement, calibrated to indicate sine-wave RMS amplitude, will only be accurate when measuring pure sine waves. If the waveform of the voltage or current being measured is anything but a pure sine Wave, the indication given by the meter will not be the true RMS value of the waveform, because the degree af nevdle deflection inan analog meter movement is propartional te the average value of the waveform not the KMS. RMS meter calibration is obtained by skewing the span of the meter So that it displays a small multiple of the average value, whieh will be equal to be the RMS value for a particular waveshupe and a particular waveshape only. Since. the sine-wave shape is most common in electrical measurements, it is waveshape assumed for analog meter calibration and dhe small multiple used in the calibration of the meter is 1.1107 (the form factor x/2 divided by the crest factor 1.414: the ratio of RMS divided by average for a sinusoids! waveform) Any’ waveshape other than a pure sine wave will have a different ratio Of RMS and average values, and thus a meter calibrated for sine-wave voltage of current will not indicate trac RMS when reading « non-siqusoidal 14 Basics uf Electrical Engineering wave, Bear in mind that this limitation applies only to simple, analog AC meters not employing “True-RMS" technology. A sinusoidal wave continuously changes in magnitude and reverse It direction at regular interval. So it is possible to state the value of AC gs simple as DC. So for this, we take four different values for indication of AC. These are: (ib instantaneous value (ii) maximum or pesk value ii} average or mean value (iv) effective or ms value 4.41 EFFECTIVE, VIRTUAL OR ROOT-MEAN-SQUARE (RMS) VALUE OF AN ALTERNATING QUANTITY The effective value of an alternating current is thal value of direct current which when applied to 2 given circuit for a given time produces the same expenditure of energy as produced by the alternating current whew flowing through the sume circuit for the same period. [1 is also known as virtual or RMS valuc of alternating current. The effective value of an alternating voltage can also be defined likewise. RMS value or effective value: jue of an alternating quantity may be determined by the ng two methods {a} Integral method (b) Graphie me For non-sinusoidal waves, graphic method is more suitable, hod 4.444 Integral Method Let /(1) be any function then the RMS value of function f(0) is tao NOgas > i tisha m4 + The rms (or effective) value of an alternating current is given by that steady (DC) current which when flowing through a given ckt for a given time produces the same heat as produced by the AC when flowing through the same ekt for the same time. + Ifa periodic current / = J, sin oxt Mowing through a resistance F. + Instantancous power absorbed by # isp prFR Aernating Cornent Fiendasentals avd Civewid 15 + Average power absorbed over a complete eyele or time Tis P, . Lf at qT, * Fora sinusoidal wave Fo J, sin ot 2 ‘ “| re sstonar] (cas 22.4] . sin 2u¢=sin 2.2 nf =sin 2.7 {on erosn2 ann. when f= {sin Jor = sin dx = 0 * Ratio of maxinrum value to the RMS value is known as eresd or peak factar or amplitude factor. + Ratio of effective value to average value is known as form factor f RMS value form factor = Average value 16 Basics of Electrical E 1 + ror) : 5 0.637 \ RMS Peak Value | Vaive w Value 1.11.2 Graphic Method In Fig. 1.19(a) a positive half cy ke of an unsymmetric: alternating current is, A r shown, Divide the period 7 ins n equal intervals of time — seconds. Let the a of current in the intervals be fy, fy, fy. mu dye IPR be it through which varying current is passed, then: instantancous middle valu the resistance of the Meat produced in: Ist interval = Fig. 1.19 7 Aernating Cornent Fiendasentals avd Civewid r 2nd interval = 2 R— watts rd interval = 7 watts ath interval = 2 a watts «Total heat produced in the interval 7 = EAB HE SATE er wants “ If is the effective value of current, then total heat produced in time T = PRT watts a) ounts of heat produged as given by equations (A) By definition the and (B) should be equal, Alternative procedure: Ifa curve is plotted in i* against time as shown in Fig. 1.19(b), then Area of Ist interval ~ Area of 2nd interval ~ i] — 18 Basics of Electrical E Total area ODMNT _ Time interval y= {Total area ODMNT ‘Time interval Thus, effective value can be determined graphically bY taking out the square root of the average of total area ovcupied by the square values of the alternating quantity in a given interval Hence, from equation Similarly, the effective value of an alternating voltage is given bY, IF the alternating Voltage is sinusoidal, then effective value will be given mating vollage. 4412 AVERAGE VALUE The average value of an alternating current is given by that direct current which transfers aerass any circuit, the same charge is transferred by the given alternating current, 1f/(0) is any function, th n average value af (1) is 1.12.1. Integral Method In case of 4 symmetrical shemating quamity, the average value over a complete ¢¥ele is zero, So the average value is obtained by considering éalfa mmettieal, the average value can only be found ng the whole e¥cle eyele only, Ifthe wave is un: out by conside For a sine wave, instantancous value is given bY, i=J,sinO Aernating Cornent Fiendasentals avd Civewid 19 Consider small interval 4@ as shown in Fig. 1-20, If'i is the average value of current in the interval, then area of elementary strip = 6. Total area of half cyete = fice Hence, the average value of current is given by wo ways. Fig. 1.20 area of half evcle w Lea interval Jiao x j i,sinad@ Y. Similarly, for alternating sine voltage £,, = 2 20 Basics af Electrical E 1 (i) hae zie "3 J Jp singw att w= Inf= 21, T=te [eos m~ cos 0 2m 1.12.2. Graphical Method For an unsymmetrical wave as shown in Fig, 1.19{a), area of curve fyb bat ti) (+ teed t Ie = T iti tiptn ti) 4.43 FORM FACTOR The form factor is defined as the ratio of the effective value to the averaye value of'an alternating quantity, Aernating Cornent Fiendasentals avd Civewid 21 For a sine wave, fective or RMS value Form factor ‘Average value 1.14 PEAK OR CREST FACTOR I is defined as the ratio of peak value of the effective value of an alternating wave. Fora sine wave Peak value Peak factor = SE RMS value hy = = 1414 0.707 4115 OPERATOR / valtage or current is a phasor quantity, but si + An alternati ee the lantaneous values are changing continuously, it must be represented by a rotating vector phasor. inst +A phasor is.a veetor rotating at « constant angular velacity. + jis defined as an operator which turns a phasor by 90° (CCW) without w the magnitude of phasor fa 12907, jrer 290° cha 1.16 CIRCUIT WITH PURE RESISTANCE ONLY A pure resistance is that in whieh there is ehmic voltage drop only, Consider a circuit having a pure resistance R as shown is Fig. 1.21 below. red Fig. 1.24 22 Basics of Electrical Let the instantaneous value of the alternating voltage applied be, E,, sin wt The instantaneous value of current, oF en =2= sin wi f= J,,sin wt E, = SM = Maximum value of current. where, The applied alternating voltage Fig. 12046). If. nd the current are in phase as shows in ! and Fare the effective values of voltage and current in the citeuit, then E=ER The voltage diagram representing current and shown in Fig. 12a) se vectors in phase is Power: Instantaneous power consumed in the eireuit ei Bog dy Si? Wt cos 2or = cos 2 ut es that instantaneous power is not fixed but has one fixed component and other compancat is pulsating component of twice supply frequency Ende For a complete evele, the average value of = cos 2 wor is zero. Hence, average power for the whole eye! Bul P E, ‘2 v2 er Where, 6 and / are effective values or ims values of voltage and current Aernating Cornent Fiendasentals avd Civewid 23 Hence in pure resistive circuit (i) Current is in phase with the voltage (ii) Current f= £ where [, E are RMS or effective values Gili} Power in cireuitis P= MI= PR 4.17 CIRCUIT WITH PURE INDUCTANCE ONLY A. pure inductive induetance and no resistance or capaeitanee as shown in Fig. 1.22. When an alternating vollage is applied ta it, a hack emf of self inductance is induced in it. As there is no ohmic resistance drop, the applied voltage has to oppose the self’ induced emf only. So-the applicd weltaze is equal 3) and opposite to the back emf at all instants. Sint Feuit possesses only Let the applied voltage Fig. 1.22 = E, sin et a instantaneous value of self induced emf is ¢” di poe di integration constant will cancel out from both side 2) 41, =e ofan observing (|}andl ¢2)-we find that the current lags the applied voltage by 94° or ® © radian 24 Basics of Electrical = impedance z By 8 * ej eaes The quantity eis calle inductive reactance and is usually devoted by symbol X; and units is chm X,,> ol ohms where, £ is in heney and @ is in rad’sec, Wave diagram and Phasor diagram for Pure “e Ein wt we Ls 1, sin (wt 2) ‘3a _j3e Fig. 1.23 Average Power—> Perdovn Fe | a sinwtty snl ne E] te aed 2 = ale! ssin we.cos w1.d(w!) SMF att) Aernating Cornent Fiendasentals avd Civewid 25 This shows power consumed in purely inductive circuit is zero. Hence, the average power consumption in an inductive cireuit is zero and is periadic with twice the supply frequency as expressed by equation (1). The stored cnergy in the inductive circuit in one quarter of a cycle is. released in the next quarter. 41.18 CIRCUIT WITH PURE CAPACITANCE ONLY When on alternating voltage is applied across a condenser, there is a regular flow ofeurrent through it, the condenser is charged and discharged alternate in cach quarter of a eVele. Consider a condenser having a capacity of C farads cont J across an alternating voltage as shown in Fig. 1.24, a Instantaneous voltage across the condenser, Ein wt e= £, sin wr Fig. 1.24 ogra tin Veo Lol, sia gut * 22) ora Jawa fae mk f | gor ? J +e (a) Wave saga (b)Phasos alagram Fig. 1.25 If this voltage causes a current to flow through the condenser whose instantaneo then quantily of electricitY stored in time df is given by, es value is 26 Basics af Electrical E = Ceo E,, cosor = cot si(or+®) = in( or) 2 ac 2 Comparing equations, we see that the current leads the voltage veetor by 90° as shown in Fig. 1.2: Maximum value of current is given bY. E ca ‘The quantity Ceo ix called inductive capacitance and is usually denoted By.X,. Its unit is ohim, 1 ” X= =~ ohms Co in farads where, © = Capacit (2 = angular velocity in rad/see E,, E0/ V2 Impedance ° 1, 290)2 = == 2-90" Since Average Power instamtancous power P = vi Po Vy sin tay Sin {oe nial ) = V1, sin wr.cos ax 0) a7 Aernating Cornent Fiendasentals avd Civewid sin 2oot-d (ar) an =0 od in purely capacitive circuit is zero. This shows that the power consun A capacitor receives energy during the first quarter c¥ele of voltage und same ducing the next quarter eyele. returns th 4.19 CIRGUIT WITH RESISTANCE AND INDUCTANCE IN SERIES Consider circuit of Fig. 1.26, Li A= Resistance in ohms in the sins: 4. = Inducianee in henties Inductive reactance = ol. E = Effective value of applied emf [= Effective value of current in circuit Voltage drop across resistence, Fj ~ RD in phase with current vector us shown in vector diagram of Fig. 1.27. Voltage across reactance, E,, = Leal. = LY, 90° abead of vector! fi oF Epeaimt Ise wie oo R [aod en en en nwt Fig 1.26 fia tar ZR = (RAT Zan Se g here 28 Basics af Electrical E and Xs R hence in R-L circuit current lags the applied voltage by angle = tan The applied voltage is therefore given by. : e-(B+G Te = YURY +0 * b=1JR 4x} = gata! Me ogy) Ob z on The quantity it +X7_ is called impedance. Since, the power is consumed by the resistance only, so the power in the cireuit is given by, PH ER=LIR R on P=EI5 If is the angle between F and J, then, cos 6 = Aernating Cornent Fiendasentals avd Civewid PH Elcosg Cos dp is called the power factor of the circuit, Obviously the power factor is lagging in an inductive circuit, Soi intancous current across RE is Je d,,sin (or ~ 9). 1,20 CIRCUIT WITH RESISTANCE AND CAPACITANCE IN SERIES Consider circuit of Fig. 1.28 Voltage drop across resistance, E, Jit in phase with # as shown in veetor diagram ofa Fig. 1.29. 1 Eo= ha co 1X, 90° lagging with respect to the current vector, me oy eR © BR vist s * Pe, a ae R L a ‘ Bok De Bre Be . E, sino Fig. 1.28 Fig. $29 The applied voltage is, therefor. fee b= JEL+ ER SYR 4X2 = 12 Thus, shm’s law is applicable to AC circuit also after resistance by impedanee. g the term Power = EF cos (1.53) 30 Basics of Electrical E ze (REX ten (=) Z- |Z) 2-9 here g=tan” BLUE Z \2|2-0 E 1=|£) 20 =1,.20 l; ° uous valtie of current throw RC is i= 1,,sin (or + 9) hence current leads the valtages bY aug 1.21 SERIES R-L-C CIRCUIT heat ' a Ee R c E Fig, 1.2918) Problems on alternating current circuits can be attempted easily by using j-notation : em 4 JIN," Bi, WHIN C= B. « Net voltage across them = + f(y — Xe) =j(E, = Bed Voltage across capac! Resistance drop = AR = Ly. © Applied voltage in /-notation is represented by. B= IR +f, - Xe) Aernating Cornent Fiendasentals avd Civewid 34 or, paifR +a, ter Impedance in j-notation may be written as, Z=R+ ji, - XO or, Ze 2o=|2|26 where ° = ix, >xcthen pissve if, > Xe then gis ve En sqe ! fnew fe 2 hence if) > Xp then current lags the applied volt fai e-$ Phasor dlagramn af series 32 Basics af Electrical E R-L-C Ckt taking £ as a refrence phasor when X; > X,- Ea Phator diagiam of a seties R-L-C Okt taking ourrent / as a retrence phasor 1.22 POWER IN AC CIRCUITS + When the current is out of phase with the voltage the power indicated by the product of the applied voltage and the total current gives only what is known as apparent power and measured in volt-ampere + Power that is retumed to the source by the reactive components in the circuit is called reactive power and is measured in vadt arin, » Power that ly used in the circuit (dissipated in resistance) is true ‘or active power and is measured in watts or KW. 1.22.1 Active and Reactive Power Form Fig. 1.30(a) N= |2| 2O=|Z\cos $+ j|Z\sino Impedance Ising Fig. 1.30 33 Aernating Cornent Fiendasentals avd Civewid Magnitude or amplitude of impedance, l= (Rex? = Power factor of the circuit Current in the eireuit This current has two companents J cos and f sin @. The component f cos @ is called in phase or wattfull componen! and J sin g is perpendicular to E and is called |, a8 shown in Fig, 1.30(b). Then Active (Real) Power = Voltage x Current x €0s gy watts less compane Since, the angle between the voltage and the wattless component of current is 90°, hence the power absorbed by this component is zero. ‘The power is only absorbed by the wattful component The total power £¥ in volt amperes supplied to a circuit consists of two components: {a) Active power = El cos watts (b} Reactive power = £/ sin $ volt amperes reactive or simply VAR. The above components can be shown in veetor from ia Fig. 1.31(2 a 8 va “\¢ “ ? ware rua, g a | 4 2 ao . ie , R Wats 9 A of Ke © (3) Fig. 131 From Fig. 1.31(b) Ot = Active power = Ef cos ¢ presented by watts AB™ Reactive power = £1 sin @ expressed by VAR OB = Toral power = Ei expressed by VA Obviously va = fWatts® + VAR (LA) 34 Basics af Electrical For practical purpose, it is more usefel to express these quantities in kilos, nce, dividing by 1000 throughout we obtain Kva = (KW? = KVAR? wo(eS) In Fig, 1-31(e) O14" Active power = ELESS expressed by KW El sin Aa’ Reactive power = “Ty ® expressed bY KVAR. O'8" = Toral power = =X expressed bY KVA " wooo “*P ™ 2 RejX FZ FR HEX (Apparent power) = (True power) + /Creactive power) Trucpower UR ‘Apparent power 127. = cos 8= Fe power factor = The apparent power in an AC CK has been described as the power the source “sees”. As far the source is concerned the apparent power is the power that must be provided ta the cieeuit => Tre power is the power actually used in the eircuit = The difference between apparent power and true power is Wasted be in ceality, only true power is consumed. Ideal situation for a system is that apparent power and true power to be equal means PF = |. This situation can occur ifnet reactance is zero. rause, 4.23 TO FIND ACTIVE AND REACTIVE POWER IN J-FORM. Consider vectors Od and O8 as shown in Fig. 1.32. Let =a, + jy = Od B and, OB / O4= ay + bP be OB= Jas + a5 + 4, b, Nl cos 0, =~ ssin = c . ‘| glo? 48 a? +87 7 Fi. 1.32 Aernating Cornent Fiendasentals avd Civewid 35 ei 83- Ga Active power: O4.OB cos AOB = Ja? +62 fai +h? - cos (8, ~ 04) 20s 6,.c08 8, + sin O,.sin Oy] 4, b, Vere Jaen = tidy + byby Reactive power: = O4.08 sin AOB = +83 sin (8, 8) sin 8,.cos 8, — cos Oy sin O)] 7. ap) bs = fae emake Jer lear = ayby—,bs WiLL = (a, + fb)) fay * fb) Keyay ~ 8,5) jlaghh + ab) te I’ Conjugate of F = ay + Jy) (1g Pb) = (ity + Bydtsh + flerjby — Bag) yay thd) + jlayby— bya) (Active power) (Reactive power) Note I: Hence, the active and reactive powers would be given by the real and j ports of the vector product of voltage with the conjugate of the current vector, Note 2: Active power can also be expressed by the sum of the algebraic praduct of the real parts of the current and the voltage and the algebraic product of the j parts of the current and voltage. Alternate approach—» Let E and J are the phasors given by Ew E20, 36 Basics of Electrical E and 1226, fae for leading current ~ sign for lagging current there complex power is given by S =F fT = 2t0, SeExd : sl 230, = ELIZ 50, = El 28,3 S= El cos (8, = 8) +f EF sin (0, = 3) S=P+ 7a if F is the refrence phasor 0, = 0 S= El cox @, + El sin (3 8) Sr El cos 0, Fj EI sin By —ve for leading P.f load S=P-j0 +ve for lagging P.f load S=P+j6 P= active pawer 0 = reactive power 4.24 PARALLEL AC CIRCUITS Parallel ckt can be solved by the follawing methods (3) Admittance method (ii) Veotor methad (iii) j method or symbolic method 1.24.1 Admittance Method The reciprocal of impedance Z is called admittance and is denoted by the symbol Y. : re 16 z (1.61) EZ ys 2p the impedances connected in parallel, then equivalent impedance of their combination is given bY, 1 1 1 = +5-+ ! + (1.61) Aernating Cornent Fiendasentals avd Civewid 37 on. Y=¥+¥ The impedance Z has two components resistance & and reactance ¥. Admittance has also two components, the conductance “g’ and suceptanee *6" ‘The impedance and admittance triangles are similar as shown in Fig. 1.33. " (1.63) x a y b oe ao R a Impedance tergle Aamittance tant (a) ib) Fig. 133 From Fig, 1.33¢h), conductance is given by, gn Yooso Since, and, (1.64) 1.65) Ify ips te CUA LO gy + Jy, yt flbgs ay Jy. the 8g) tgs+ e+. mho (1.66) 38 Basics of Electrical E mho vf 1.67) (1.68) (1.69) Total current 1.70) Power factor angle, o-unt & (1.71) g Power factor will he lagging if} is + ve Power factor will he leading if is - ve Note: Inductive suceptance his as ~ve sign sned + ve sign and capacitive suceptance 4.24.2 Vector-method Considera parallel circuit shown in Fig. 1.34(a) ' rE . E . @ Fig. 1.34 Branch I. (1.72) 1.73) (1.74) Aernating Cornent Fiendasentals avd Civewid 39 Take E as reference vector. Draw J, lagging at an angle 9, with Eas shown in vector diagram of Fig. 1.34(b), Branch 1. = (Ri +x = fate? (1.78) Impedance “ i va(1.72) 9; stan" Re leading 1.7) Draw J; leading £ by 9). The resultant of J, and f, will give total current £ and the angle between £ and will give the pf angle: Thus, a parallel circuit can be solved easily in this way, 1.24.3, j-Method Consider a parallel CKT of Fig. 1.34, we can express the various impedances inj form as under. Zo Rye pX wk Ry -jXe te | 2, Rt ix, AR + ie — CR, + Ra AN, ge ER NY HY ay NR R +R) “OK ~ XO) RAY HO, KP (B+ RP HOY, Z= RON 2. Total current drawn = = Zz Power factor cos. = Power factor be tay leading if ¥ is—ve ging if. is +ve 40 Basics af Electrical E 1.25 RESONANCE An AC ckt is said to be in resonance when the applied voltage and the resulting, current are in phase. Thus, at resonance the equivalent complex impedance of the CKT consist of only resistance. 1.26 SERIES RESONANCE OR VOLTAGE RESONANCE Look L ate ae Fig. 1.38, = ! +. R jl - } = Vand J will be in phase when reactanee is zero. => air on ce The net reactance is zero, Ifthe resonant frequen we 2h, => Series CKT at resonance termed as acceptor CKT, since, it accepts frequencies clase to resonant frequencies when impedance is minimum, = al low frequency A> Nand the CK factor is leading => at high frequeney 4, > Ae and the CKT is inductive £6, < and power factor is lagging js capacitive tf, fF) and power = as f goes on creasing impedance also goes on incre: ing => if the resistance is zero, at resonance the CKT acts like a shart circuit Aernating Cornent Fiendasentals avd Civewid a voltage across capagitange K c naximum can be obtained —“ = 0 dw => it has been found that at resonance the values of I’, and Fy-may be higher even then the supply voltage at resonance Fy = Mo. , Phasor jagram at resonance — aM Ws, Fla, 1.36 42 Basics of Electrical 41,27 BAND WIDTH Band with ofa series CKT is defined as the range of frequency for which the power delivered to the resistance is greater than or equal to half the power delivered at resonance. une, " y= Current at ressnance b > \ Less R R \ eee Xf oes High R fp tf hot & (Reanance curva) frequency Fig. 1.37 Curve between current and frequency is known as resonanec curve. Band width =~ f = wy my wy andy re the angular fre power delivered at resonan: jencies al which the power delivered is half the . These are also known as half power frequent I => At resonance Z-7ek ; _ r Athalf power point Z= p= 7 & 2 Za (Rae? RN2 = [Rt +X? so at this point x= R => Lower half power frequency w) XeoX,=R Aernating Cornent Fiendasentals avd Civewid 43 oE were tlt ve frequency is meaningless so we take only +ve frequency Atupper half frequeney w, p= X= R Band width = amd. yy = 1.28 QUALITY FACTOR AND SELECTIVITY Ratio of resonant frequency to band width is an indication of the degree of selectivity of the CKT and this is known as Quality factor, @. or, o=- —“h__ Walt, — 65) mi") wWyeR a 44 Basics af Electrical E Higher values of @ (4) Resonance curve is very narrow and sharp (£2) R > Sharpnessaf the curve depends on the parameters R and 2. BY changing C the resonance can be made Ww oveur at different values of irequeneies. total stored energy ‘energy dissipated/eyele wk Qeeb R aeVLlC-R wL aw 8 o- Selectivity = => ACKT with 2 flat frequency response curve (high R) will be more ponsive and therefore less selective at frequencies in the neighbourhood of the resonant frequency. 1.29 PARALLEL RESONANCE A parallel combination wf &, Z and C ot (®, L) and C branches connected to a source will produce a parallel reser nntisresonance) when the resultant current through the combination is it phase with applied voltage at resonance power factor is unity for this. 45 Aernating Cornent Fiendasentals avd Civewid 2% TT voppedea @ yl joyared U) parsouvos st Y eouRysisG! soyas angooya pur J aaueNpUE Jo BOO y (pneu yue3) prove yoqeved jeaqoesd ge TAA oy , TM TH comm « °M = M ‘0 = SoUBUOSGL Ie Gora auoz sig weg | 7 fuouBeus sovevosas ie TMA 2H al p, HEMP | Y aeons em uTE att a - ary SOI = WY oy Bak ' L Ke ha the A Oo pues (7 yy) ore UNDO “TY PHB 1E AONANDAYs FONVNOSSY NOS NOWLWAINAG 0'L Basics af Electrical E 46 "Mt = M0 = geouEUOSeN Te oa) Wonk Tk + ou y =A you UE} Se WaroUy 5} LOREUCWAD 27 jee = noua Jeunsou joUeved JPA puczas # PAllED ‘O pue “Ty FalleTed Jo BugSIsU09 LOW = ‘SQUEUOSO! YO UIMWRALY 8) OY 1-8 4s canna i enue mb 8 ware 21 gosjpe 27h coy 4 Aernating Current Fiendamentals and Cinewit a7 1,31 IMPEDANCE AT RESONANCE At resonance w = At resonance Z,,= Resistive part Z,= Ryis dynamic resistance -n (4-8) RUC FE RC. Z, is called dynamic impedance, this is pure resistive. It is seen lower the & higher the 2, Hence the value of impedance at resonance is maximum and the resullant current is minimum, 4 parall ejector cireuit since the current at reson: rejects the current at resonance. lan resonant circuil is also called a eis minimum or tank circuit almost 1, = Current at resonance it R= 0 cht will draw no current £ z. zero and large current circulates in parallel at resonance, The supply curren ckt at resonance. 4.32. CURRENT MAGNIFICATION Current drawn from supply at resonance is J = PCR on [I= OF T 48 Basics af Electrical E 1 mh te wea aR -iwe EL Fig. 138 So circulating current is I'w,C. = —Liteuloting current . Coment drawn from suppl’ jc. B L = Parallel tuned eireuil exhibits a current amph okt exhibits voltage amplification of @ on of @. whereas series 4.33 SELECTIVITY AND BAND WIDTH At half power frequeney wr, and wy. ckt impedance is . {Atreasonance J’= J. R, {Athalf power F= LR Bond width = wy — i, 1 wWCR Comparison between Parallel and Series Resonance (1) Net suscemtance is zero while in series resonance reactance (net) is zero (2) Admittance is equal to admittance at parallel resonance. (3) Reactive or wattless component (/ sin @ — f,) is zeto, hence circuit power factor is unity. L 4) Impedance = — while in series resonance Z = (4) inp: Gap Mile in series resonance Z= R Aernating Current Fiendamentals and Cinewit 49 V {5} Line current is minimum and equal ta > while at series resonant it ts rmaximurn and J 5 soseries resonant ckt is acceptor while parcel ckt 1 ic 1.34 CURRENT AT RESONANCE ‘The current flow ing in the cireuil at the time of parallel resonance is minimum. For circuit given below, equivalent conductance is, is rejector FeV Eee HZ a in erie cht, (6 L, f= hh | = mi 5+ fe : (1.82) Re RNP RE+ NG A ve) SR IPR, = Ry =O, then oe e g-0 Also susceptance, b=by+by x, eos (1.83) Rtn; “ = Oat resonance 2+ Equivalent impedance, i 1 ze = neo (1.84 y o (sh) Current = (1.85) Such a circuit is called a rejector circuit, Tn actual practice, the inductance coil may have a small resistance say A and the resistance in the capacitive branch R, be assumed zero, then R Rex? Total conductonce of z = 7) 50 Basics of Electrical E TER is small cnough, whose squares may be neglected, then (1.86) SOLVED EXAMPLES Esample 1: At clement series circuit is connected! across an AC source V= 300 cos (F141 + 20°) volts. The current is drawn 15 cos (M4r— 10°) Amp, Determine circuit impedance magnitude and phase a power drawn? ale, What is the average (U.P. Tech 2003-04) Solution: Given, Vr 300 cos 3146+ 20°) [cos @ = sin (B+ 90°) = 300 sin (3142+ 110%) In polar form = 2 z11u v2 os (314e = 10) 13 sin (314r-+ 80°) Is . feo eon 300 100 v2 Z (circuit impedance} = 20 230"| Henoe, the angle between voltag and current is 30° and current lags ) the voltage by 30°. Phase angle = 30° casi (in Reb ckty 615.4300 x cos 30" = 1949.85 walt Aernating Current Fiendamentals and Cinewit 51 Example 2: A 120 V, 100 W lamp is to be connected toa 220 V, 30 Hz AC supply. What value of pure inductance should be connected in series in order to run the lamp on rated volta (2003-04) Solution: => Vasguy #220. 50 He Fig. 1.39 => Current through the lamp and inductance is same. Current through Lamp = 120 x F, 120 = -— - — 39 = 221.269 > 100 100 x LB. 21.269 ohm. ” X= 2nfL = _ 21.269 Daf 2x3,14x 50 = 0.7046 henry 52 Basics af Electrical E Example 3; For the cireuit shown in figure, find the current and power drawn from the source. (2004-05) sa iH oe ote tee a oa 8 220, S02 Fig. 140 Solution: Let Za34ja=$253130 Zz =64j8=10.253.130 Lye dy 94 jl" 18 £53.13 Both Z, and Z, are parallel hence net impedance of the circuit is Z ga Bee. 5x10. 2106.26 242. 15 253.13, Z= 333 253.13) => Current drawn from the ckt is P= Fo 230 ZA 33325313 [= 69 Z~ 33.15 Amp) Hence, net current lags the net voltage by 53,13° and circuit is inductive in nature = Power drawn from source = If cos @ = 230 x 69 x cos ( =9.522 kw Ans. 13) Aernating Current Fiendamentals and Cinewit 53 Example 4: A coil connected to 100 V DC supply draws 10 Amp and the same coil coanected 100V, AC voltage of frequency 50 Hz draws $5 Amp. Calculate the parameters of the coil and power factor. (2004-03) Solution: => Coil means a resistance and inductance both. Let impedance of a coil Z= R +); => When DC supply is connected to coil inductance behave Tike a short efreuit (X= 2nfL = 2n x Ox L= ON) So resistance oF eoil R=“ = ie = ohm, > When AC is applied acrass the same cail, Given = 190 voltof 50 Hi frequency 1-5 amp = > =o = => 2-1 x, ~ V300 = 17.320 be Aes 72 05 hemy Inf 2x3.14x50 a => Power factor of coil = © ~0.5 Ingging Ans he current drawn and the power factor in RLC series circuit, a series RILC circuit with R= 100, 1 = 0.02 Hz, and € = 241f is connected to 100 V variable frequency source, Find the frequency for whieh the current is maximum, (2004-05) Esample 5: Discuss the effects of varying the frequency upon Solution: Fig. 1.44 54 Basics af Electrical Impedance ZR +N, —JXe Z=R+i4,—Xo) Rea ca 1 Ze fe, -X OF Ztan te = 17) 20 r Is \Z<0 Re + (8) =X and PE cos (when ¥, = Aes > z ce frequency J resonant freq (f) = |2|=Rso current is maximum and power factor is unity: (2) now if we increase 4xc= —|__ will decrease and 4, increases, Impedanez increases hence Dae current will decrease and power factor de. G3) If frequency decreases below resonance frequeneY (f'< J, then Xp decrease and ‘Xj increases but net impedance will increase, so current will dectease and power Factor will also decrease. the frequency} from resonance frequency f> f,. Then ses andl becomes lagging. & cos 9 = = but it becomes capacitive => Current is maximum at resonance so at resonance frequencY Aernating Current Fiendamentals and Cinewit 55 f= \ = 795.5 Hz Amy 2% IO" x 2x LO +A load having impedance of (1 + 1}.Q is connected wo an AC Example voltage represented as # = 20.Y2 eos (wi + 10°) volt, Find the current in load expressed in the form of f= f, sin (wr + @) A. Find the real and apparent power. (2004-98) Solution: Load impedance Z=1tsl= V2 245° Voltage across the loud J°= 202 cos (wr + 10°) = 207 sin Ger + 100°) 202 y= 22 rou = 20.2100" Se A100" = 20 2100 Current through load p= E = 20¥100 v2 24s? T= 144 2354 is rms value of current J, sin (wt + Oe fy, ()a i= V2 Lin (ws + 55°) 20 sin (wi + 55) i (ii) Real power = Figg yy, CO8O 1,008.0 Ler 20.008 45 P= 200 wait 58 Basics of Electrical E Gin) Apparent power = = L208 x20 282.89 VAR Example 7: An em en by 100 sin [sur- ") volts in applied to a circuit and the current Gi) «i 20 sin (314¢ ~ 1.3808) ampere. Find (i) frequency uit elements. [2005-06] Solution: WL instantaneous emf be ¢ = 100sin (s-3) 4 = wi= 3141 => Qaf=314 = f=50He 0 (ii) e-! wr 4 1 = 20 sin (314r— 1.5808) 5 = 20sin (a1 eet) Bt = 20 sin (ser > Is v2 Ve- Par) Circuit impedance Z = fé- wit Fig. 1.43 Aernating Current Fiendamentals and Cinewit Ss? 100 = Current lags the voltage by ‘ So circuit elements are & and £. Z~R4jX,~S cos 45 + j5 sin 45° R= Seosase= 2 a v2 Xp oSsinase- 0 => Ap = 2Rfl = po Xe — = 0.01126 henry, “Inf Jax2xR x50 Example & A choke coil takes current of 2amperes lagging 60° behind the applied voltage of 200 volts at 50 Hz. Culeulute inductance, resistance und impedance of the coil, Also determine the power consu connected across 100 V, 25 Hz supply. (2005-06) ed when = 22-80 4 200 20, 50H Fig. 14d Coil having a resistance and inductance both ¥ 200 => Impedance af the coil = — 7 22-00" 58 Basics of Electrical Z= 100 £-60°= Re jX, eke is inductive) 100 cos 60 + 7100 sin 60 = K+ 7X, > B= 100-c0s 60 = 100 x t =s00 v3 A, = 100 sin 60 = 100 86,6 82 => Frequency of supply = 50 Hz x, _ 86.6 2nf 34 2.758% 10H (ii) Now the choke cail is connected (0 100 V, 25 H Rand £ will be same as above, aly. Now, My = 2efl = 23.14 25 2.758 x 10+ = 43.3.0 : > 43. Now, PNP = 50) $43.3? = 66.1 Zion! = 0 Za JR + + tan = 66.1 £40.89 y vd) current from the coil = > and current from the coil = 5 100 f= ———_ = 5 2-40.89 Amp 66.1 240.89 Power consumed = Vi cos. = 100 15 x 0.75 = 112.5 W or, PR (SFX 50> 112.5 W Example Two coils of $Q and 10.0 and inductances 0.04 H and 0.05 H respectivelY are connecting in parallel across 200 V, 50 Hz supply. Caleulate (i) Conductance, susceptance and admittance of each vail and its power factor, (ii) ‘Total current drawn by the eire (ii), Power absorbed by the circuit, Aernating Current Fiendamentals and Cinewit 59 ‘The value of resistance and inductance of single coi] which will take the samme current and power as taken by the original circuit (2005-06) Solution: Given: "oe 10:0 I ws a b 200, SOHZ Fig. 148 (i) Ae RAEN, N,, = 2nfly ose inse~ 1382 2682 = 2x 3.14 502% 0.04 2,5 $+ /12.56= 13.52 £68.29 10860 By Rt iX,, Niy= afl = 2x 3.14 «30 x 0.05 = 10 +J15,7 = 18.62 £57.51 =157Q Admitianee of ¢ 11) sy) = G +78) = 1 = = = 0074 2-68.29 13.52 268.20 = ¥, = 0.074 2-68.29 ¥, = 0.0274 — 0.069 > G, = 0.0274 and susceptance #; = 0.069 Admittance of coil (2) is Y, = G, + jy Yy= = A = 00537 2-57.51 18.62 257.51 "= 0.029 - f0,0453 60 Basics of Electrical E So conductance G, = 0,039, susceptance B= 0.0433 Gi) Total adenittanee of ckt is ¥= 1, + ¥ ¥ = 0.0274 — 70,069 + 0.029 — 70.0453 0.0564 — 0.1143 Y=01275 2-63.04 Curtent drawn bY the circuit is #= VY 1200 « 0.1275 4-63.74 Heace, current lags the supply voltage by 63,74". So pawer factar = eas (63.74) = 0.443 Gi} Power absorbed by the cireuit P= feos 6 P= 200%2 10s (63.78) = 2.256 KW Ans, iv) Current taken by ginal circuit is 7 Fe 200V 25.5.2- 63.74 Amp : F200 Impedance of coil Z= K+), = = i 25.5 £- 63.74 2=7.843 263.74 2347+ /7.0M So R= 3.47 Qand X, = 7.034. Example 10; An AC vohage e(f) = 141-4 sin 120/ is applied 10 a series RC circuit. The current through the cireuit is obtained as 4) = 14.14 sin 120) + 7.07 cos (1208 + 50°) (2004-05) Aernating Current Fiendamentals and Cinewit 61 Determine: (i) Value of resistance and capacitance (ii) Power factor (iti} Power delivered by the source, Solution: (0) = 141.4 sin 1201 time domain M4bd “EF Given, ag) = 14,14 sin 1202 + 7. 14.14 20+ 7.07 sin (1207 + 120°) 14.64 20 + 7.07 2120" 14.14 + 7.07 cos 120 + j7.07 sin 120 E 20° — polar form 03 (1207+ 30) cos @ n (0° +0) 14.14 - 4.535 + 76.123 HE) = 10.61 + f6.123 = 12.25 229.989 ae) =12, (1207 +305) _1n25 1 230° a v2 Y 12.25 v2 2= 9.996 -j5.17=R > R=9.9960, Vo=5 @ z 230 = 11.54 2-30" (ii) Power factor = cos 6 = €08 3 = 0.866 Gii} Power delivered by the source cos 9 62 Basics af Electrical E 1.4)(12.25) cos 30° P= 750 watt Example Il: A non-inductive resistance of 10 ohms is connected in series: with am inductive eoil across 200 V, $0 Hz ac supply. The current drawn by the series combination is 19 amperes, The resistance of the coil is 2-ohms, Determine: (i) Inductance of the coil Gi} Power factor iii} Voltage across the evil Solution: Given ae nt Fe sona _ © 200 v, 50 H Fig. 1.48 nf Za WWF 2ejXy= 12+ /Xp= ol’ voa . - 00 og 7 10 = = A, = 4256 = 16 ohm Xp = Daf pe Ac —!8 50.9 mitt henry Inf Bx 3.14 50. Aernating Current Fiendamentals and Cinewit 63 2 =0.6 Gi} Power factor = 0 Nie {iii} Voltage across the coil is V,, then Vee IQ +iX) Vy = 10(2 +f 16) = 161.248 £82.87 volt Example 12: For the given figure shown ia i728 6a -joa wov 3 st 50 He 4 » z Fig. 1.47 (i) Admittance of each parallel branch ii} Total circuit impedance Supply © und ower factor (iv) Total power supplied by the source. (2005-06) Solution: Lyn 16 4712 1373 LITAT ZHA+ sR 8 236.86 Zy-6-j8= 102-5 13 (i) Admitiance of each parallel branch is ¥, and ¥;, then Sees ap gy OSS e 7375 £77.47 =0.2 2-36.86 O I «3236.86 64 Basics af Electrical Ys = 0.2 feos (-36.86)" * sin (-36.869] =O.16-j0.119 (ii) Total circuit impedance is Z = (2, | 23) + Zy 502 = 164/724 = enyra+ 80421627 11.18 2-21 1.627724 4.47 210.29 1.645 7.244.398 4 50.798 Z= 5.9984 j11S9R= 13.06 262.65] Ans. Vv 100 iti} Supply current: = = GH) Supply cument 1 7 13.06 262.68 7= 765 2— 62.65 Amp] Power factor = cos 62,65 = 0.459 (iv) Power supplied by source ~ I’Teos 6 P= 108 7.65 cos (62.65) = 351.13 watt pe or, = (7.63) x 5,998 = 351.02 wat Example 13: For the circuit shown below, determine: (i) Resonant frequency Gi} Total impedance at resonance Aernating Current Fiendamentals and Cinewit 65 (iii) Band width (iv) Quality factor <2s0 b Sat o Bos Fig. 1.48 Solutions (i) Fora pa allel ckt resonant frequency J, is given by i i 2x5.14 0.5 x5 x10 = 100.39 Hz £ ii} Total impedance at resonance is Z = — iip Total impedance at resonance is Z = 5 03 asxsxige 8 R 250 an (iii) Band width (= ,)= 5-196 He 66 Basics of Electrical E Example 14: Drew the phasor diagram showing the following voltage and find the RMS value of resultant voltage. Hy = 100 sie SOs, 1”, ~ 200 sin | $00 + 5) ¥,=—S0 eos ($000), y= 150 sin {soo - *) Solution: = If I'= F, sin (wr + 8) can be represented in a polar for I’ = [9 and shown in X+¥ plane Similarly > Y= 100 sin 5001 100 => Wy = 0 v2 x > 200 sin (5001+%) = v,- 708 2% 3 Ao = 30 cos (500%) =~50 sin {500+ ) =>, 2 = s0 sin (s001—4) =F 3 ¢ » + robes Fig. 1.80 Aernating Current Fiendamentals and Cinewit 67 Resultant veliage Veale lyt hye 100 200 ,a (=-50) x, 150 = 204 254+ <4 2 v3 (=) 2 {100+ 200cos¥ +2005 g 50+150c0s(=*) -rea(-) = L100 + 100 + £173.20 ~/$0 + 106,06 ~ 106.06] Be = Sp lias +1718) vi _ 306.54 v2 F 232,05" 306.54 RMS value of resultant voltage = = 216.756 volt and resultant valtage leads feam reference axis by 32.05°, in (wt + 6) Instantancous voltage = 6. a S000+ 5 | Example 1S: A series R-L-C circuit has R = 10.0, = 0,1 Hand C=8 pF, Determin (i) Resonant frequency Gi} Q-factor of cireuit at resonance Giii} ‘The half power frequencies 8x 10°F R-L-C circuit resonant frequency f, is given by lution: Given: @= 109.L=@.1 H.C= (ip Fora seri 1 Noa Jee 68 Basics of Electrical E he ' - 253.14 fore ss10! 24 Hz, Gi) Q-factor at resonant R LANGE 2x34 x 177.94 x01 R uy = WAT iii BW=h-f 4f; and fy are hall power freque: a = Whos =¢- =177.95- 9 __ = 169.99 He dal 4x3)4x01 = Aad an = 177.94 + 7.95 = 185.89 Hz. Example 16: An alternating current of frequenc¥ 50 He, has a maxinwra of 100 A. Calculate (ap its value second afler the instant the eurrent is zero and its value decreasing thereafter ¢b) How many seconds after the instant the current is zero finereasing therefore words)? Will the ewrrent attain the value of 86.6 A? (Elect. Tech, Allah. Univ. 1991). Solution: The equation of the alternating current (assumed sinusoidal) with respect to the origin of Fig. 1.51. = 100 sin 2m 50¢ © 1 D sin 100m {a) Hshoutd be noted that, in this case, time is being measured from point A and not from 0. If the above equation is to be utilized, then, this time must be referred to , 1 point @. For this purpose, Ralf periad i.e, [> see. has to be added to Aernating Current Fiendamentals and Cinewit 69 “ ata Ne zoe Fig. 4.84 Ll - 1 1 = sec. The given time as referred to point O becomes = — + 600 100 600 -J e 609 *** 7 b is 100 sin 100 x 180% —— = 100 sin 210°. 600 = 100% (-4) =-s9 Point B. (b} In this ease the reference point is O ‘ 86.6 = 100 sin 100 x 1807 ar sin 18,000s = 0.866 or 18,0001 = sin’! (0.866) = 60° ‘ eon " so00 300 Example 17: An alternating voltage ¢ = 200 sin 314f is applied to a device Which offers an ehmie resistance of 20.2 to the flow current in one direction while preventing the flaw of current in opposite direction. Calculate RMS current over 0 (Elect, Engg. Nagpur Univ, 1992), value, average value and form factor for d Solution: Comparing the given voltage equation with the standard form of alternating voltage equation, we find that a _ _ 200 | ¥,= 200 V, R= 202 f= > = WA 70 Basics af Electrical E For such a half-wave rectified current, RMS value = Im _10 Average current = = 1% = 3.18 As Form factor = —— = 1.57, Example 18: A 50 Hz sinusnidal voltage wave shape has maximum value of 330 V. Calculate its instantancous value {a) 0.005 second after the wave passes through zero in the positive direction (b} 0.008 see after the wave passes through zero in the negative direction Solution: @ Eq sin wt 4° 0.005 sce, £,, = 350 V. f= 56 Iz = 380 sin2xMxSOKE = 350 sin (100 x 180 x 0.005) Je = 350 volt So instantaneous value at ¢= 0.005 sec is the maximum value (b) After the wave passes zero in negative direction. ‘maf r Sine wave will pass zero in negative direction after half e¥ele or = see. Tr = 0.02 sec, rt = 6.0] sec Now, e = 350 sin (25/1) Aernating Current Fiendamentals and Cinewit a” = 350 sin (2 x 180 x 50 x 0.018) 208.72 Volt Ans, Example 19: A sinusoidal alternating current of frequency 25 Hz has a maximum value of 100 A. How long will ittake for the current te attain values of 20, and 100 A? Solution: For AC current f= 7, sin (we) Given Jpg = 100A, f= 25 Ha, #100 sin $02 {a) When current attain value of 20 amp, means instantancous valuc #= 20 amp. 20= 100 sin 5Onr sin SQ = 0.2 S0m0 = sin"! 0.2 = 115° rs? s® = 0.00128 sec SOx 50x 180° (b} When instantaneous current #= 100 amp i= 100 sin 50n7 00 = 100 sin S0n¢ > S0ne = sin’! = 90° > pe 2 9.41 see. 50x 180° nple 20: A sinusoidal current wave is given by i= 30 sin (100nd). Determine (i) the greatest rate of change of current (ii) the average value (iii) Foot mean square Value Civ) the time interval between a maximum value and the next zero value. Solutio (i} Given, 7 = 50 sin (LO0xr) 72 Basics of Electrical Rate ofehange of enrrent = -_ di i= © = 50x 1007 eos (100m) di £=1,,c0s (100R) So greatest rate of ehange of current is Py r. S5000m = 15,715 amp/see (i #= 50 sin (100K) 1, sin (viel in sin (wide fay = 0.637 J, = 0.037 X 50 = 31.85 amp RMS value for | Ly sin wt Aernating Current Fiendamentals and Cinewit 73 t fags > |= Je sated 0.707y, Joay = $0. 0.707 35.35 amp: 15 dy sin 100 ‘The maximum value will attain when #~ J, sin 100n2 = | Loon: = 90° 4 = —_ = 0,008 sec 100x180 = Aftera maximum value the zero value will come after Ist half cycle. Bok 7" 3 l 2x50 0.01 see = So the lime interval between a maximum value and the next zero. value is 0.05 see 10 0,01 se 74 Basics of Electrical Example 21: The voltage across and current through a circuit are given by Gl4t + 80°) A. Calculan V= 250 sin (314¢ — 10°) volt and ¢ = 10 si impedance, resistance, reactance and power the or of the circuit, Solution: given ¥= 250 sin (3147 — 10°) volt i= 10 sin (3141+ 50°) amp above voltage are in time don weecan write in polar form => Impedance of ckt 2235 2-60" From this it is elear that eurrrent leads the voltage by 60° So power factor ~ cos 60° = 0.5 = Zo Re JX p= 25leos 60+ sin (-60)] RX p= 12.5 —j21.65 il imaginary part Ra 1250, X= 21.650 Ans Comparing Example 22: Fig. 1.82 Aernating Current Fiendamentals and Cinewit 75 Find (a) Total impedanee (>) Current drawn from supply (c) Voltage across each branch (1), (2), 3) (d) Current in each branely (e) Power factor (0. Apperent, active and reactive power (g) Draw the phasor diagram Solution: Let 2ej3= V3 Zan 1-j5 = V26 Ztan! 7 Zaderte $20 Zan | ZZ, {a) Total impedance Z= 2) + za (as jp Lisa (l= f5)+ (44 f2) = (24/314 G63 —JL.41) %=5.65+j159=$.87 £18.7° ohm Vv 10 6 wie o Z 587 218.7 = 1.70 2-15. amp means total current / lags the applied voltage P= 10 20 by 15.7° te) Pye = LZ pe |e Ve ® 1.70 £-15.7 (3.65 -f1.41) =5.32-j4= 663 2-36." => This indicates voltage across BC lags the current by (36.8 — 15.7°) = 211°. 76 Basics af Electrical E «a vi = Me. = = 130 £41.9° 2; current fy leads the Figg bY (41.9% + 36.8%) Fac. - 685 ZO368 yy 263.48 "2 4aj2 fe) Power factor cos § = cos (15.7) = 0.963 lagging, 5.65 or, coro eB . £88 963 Z 587 (f) Apparent powers = I = 10% 1.70=17.0.VA True power = R= 17x 1.7% 3.63 = 16.32 W, = Vicos 8= 10x 1.7 x 0.963 = 16.34 W, Reaetive power = PN = 1.7 1.7 4 1.59 = 4.59 vars = Vi sin 85 10% 1.7 x sin (15.7") =4.6 vars (y) Phasor diagram Let = 10 20 isa reference, Fig. 1.89 Wernating Current firntamentals avd Circuit 77 Example 23: An alternating current of frequency 60 Hz, has @ maximum value of 120A, Write down the equation for ils instantaneous value. Roknocking time trom the instant the current is zero and is becoming positive Find (a) The instantaneous value after 1/360 second and. (b) The time taken to reach 96 A for the first time Solution: The instantnneous current equation is i= 120 sin In fr= sin 120 re. Now, when ¢= 1/360 second, then fa) 1 = 120 sin (120 2c % 1/360)... angle fn ra = 120 sin (120 > 180 x 1/360)... angle in degree = 1200 sin 60° = 103.9 A. (by 96 = 120 x sin 2 x 180 x 60 xr angle in degree on, sin (360 x 60 A= 96/120 = 0.8 “ 360 x 60% ¢= sin’! 0.8 = 53° (approx.} 1 = O/2e f= 53/360 x 60 = 0.00245 second Example 24: Ap alternating current varying sinusoidally with a frequency of 50 Hz, has an RMS value of 204. Write down the equation for the instantaneous value and find this valu (a) 0.0025 second (b) 0.0125 second after passing through 8 positive maximum value, At what time, measured from a positive maximum value, will the instantaneous current be 14.14 A? [Eleet, Se. 1 Allah, Univ, 1992] Solution: = 202 = 28.2A, Hs 2mx 50 100R redsee 78 Basics af Electrical E The equation of the sinusoidal current wave with reference te point 0 Fig. (1.54) as Zero time point as f= 28.2 sin 100m/ ampere Since, time values are given from point A where voltage has positive and max. value, the equation may itself be referred to point 4. In the case, the equation becomes 28.2 cos 100i 06,0023 second 28.2 eos Ltr x 0.0025 angle in rad. (i) When, = 28.2 cos 100 x 180-x 0.0025 angle in degree 8.2 eas 45° = 204... Point B. Gi} When, 1 = 0.0125 second 7 = 28.2 cos 100 x 180 x 0,0125 $0 =28.2-1/¥2) 20.4... point C. (ii) Here, io hla ‘ 14.14 = 28.2 cos OD x 180 / 2. cos 100% 180 f= or 100 180 F= cos 40.5) = 60°, 1= 1/300 see. ... point D. Example 25: For the trapezoidal current waveform of Fi |. 1.35 determine the eff ve value, (Elect. Tech, Vikram, Univ, Ujjain. Nagpur Univ, 1999] For O< ¢-< 37/20, equation of the current can be found from the Solutio relation, j 201 {From © to is equation at ¥= ax} rea TTi20 hy 4 we » Fig. 4.55 Aernating Current Fiendamentals and Cinewit 73 When 37/20 <1 < 77/20, equation of the current is given by i= J, keeping in mind the fact that A O48 is identical with A CBE. RMS value of current = Example 26: What is the significance of the RMS and average values of a wave? Determine the RMS and average values of the waveform shown in Fig, 1.56, (Elect, Tech, Indore Unis.) Solution: The slope of the curve 4B is BCC = 20/F. Next, consider the function yat any time ¢, It is seen that DEAE = BC/AC = 107 or, (v= 104 = 10/7 or, y= 10+ (O/T) tr eT Fig. 1.56 80 Basics of Electrical E This gives us the equation for the function for one cycle. Mean square value = Ff [ies Bee Ea r . : bong 4 Oe, How? |’ 700, T wo |, 3 or, RMS value = 10 Esample 27; The halfcycle ofan alternating signal is as follows it increases uniformly from zero at O° to f, at e", remains constant from s* to (180 — o&}° decreases uniformly from J, at (180 - sc)" to zero at 180°, Caleilate the average and effective values af the signal. [Eleet. Science — 1 Aah, Univ, 1992] Solut For finding the average value, we would find the total arca of the trapezium and divide it BY m (Fig. 1.57) 7 D fy ten) Fig. 1.57 Aernating Current Fiendamentals and Cinewit 81 Area = 2x AOAE + rectangle ABDE 2 (12) fay Xo + (RD Vig (R-=My * Average value = (=f 5 y RMS value from similar triangles, we get = This gives the equation of the signal over the two triangles OAF and DBC. the signal remains constant over the angle to (ie — s-) be., aver an angular distance of (m— e+) —e= ix — 2se) Sum of the squares = 0e0 + (2 tm Bee) =f (x “*) (aha) r L The mean value of the squares is = —f: r i RMS value = f, 8). at Example 28: Find the average value, effective value and form factor of a s¥mmetrical alternating current wave, whose half-e¥ele is sh ‘The current rises front 0 0 19.4 in one-third period, i middle one-third period and again Falls to zero at a un third period of half cycle. 1008, 1008 HON OA, of 52 2a a Fig. 1.58 82 Basics af Electrical E Solution: Average value of current area of teapezuim interval 1 x nil x x1OX +10K 2+ 10K =2 3 3/2 5 m ~6467A Anse Draw the dotted trapezium by plotting square values of current, then eflective value of current, sa of the dotted tapeziun interval ‘100% +100%% +h x10 ® 2 3 2 3 x =815A Ans, 815 ” Form factor = 7s 222 Ans. Example 29: Determine average value, effective value and form factor of a sinusoidallY var¥ing alternating current whose half wave is rectified im cach cyele, Solution: Average value of current is given by, area of reactified wave interval Fig. 1.59 Aernating Current Fiendamentals and Cinewit 83 fia tn J 1, sin a0 Te = (1.24) Effective value of current, fede [E: sin? 9d0 ys ya z “ Form factor = Example 30; Three coils of resistances 20, 30 and 40 Q and inductance 0.5, 0.3 and 0.24, respectivelY arc connected in series across.a.230 V, 50 c/s supply. Calculate the total current, power factor and the power consumed in the circuit, Solution: Total resistance & = 20 +30 +40 = 900 Total resistance £ = 0.5 40.3 +0.2= 1.0.2 * X= Impl nx 50% 10-3142 Impedance 2 = (fR?+X} = 907 +318 = 3270 Basics af Electrical E 84 L104 A. ND ONIS ‘ Current 1 ietar cos @ = Power = 0.275 lagging. Power consumed = Bf cos 0. = 230% 0.704 x 0,275. = 44.5 watts. Example 31; A resistance of 100 9 and a capacitance of 40 MF are connected in series across a 400 W supply of 50 cis. Find the current, power factor and the power consumed in the eirewil. Draw the vector diagram, Solution: R- 1000 1. igs Daf. IRKSOKADXIO Impedance Z= 1007 +795? = 1278 Q 400 * Cument = jog RIBAS Re 313 N ¥ 3, 8, | /ow X9A poy IXe= 288.5 Fig. 1.60 Aernating Current Fiendamentals and Cinewit 85 R Power factor = eas 6 = = wer factor z 100 = = = 0.783 leading. i278 783 leading. Power consumed = EF cos 9. = 400 3.13 x 0.783, Now, /# = 3.13 x 1005313 V Ne = 313 x 79.5 = 248.5 V fake / as the reference vector. Draw [R= 313 V toa suitable scale in phase with fas shown in phase with J hown in Fig. 1.60 Draw iX,.= 248.5 V, 90° logging with respect to current vector, The vector sum of A ond LX will applicd voltage E = 400 V. On measurement, the angle obetween Fand F is found 398.S°. Thus, the complete vector diagram is shown in Fig, 1.60, represent Example 32: A series circuit consisting ofa resistance of 100 Q, inductance of 0.2 H and a capacitance of 20 F is connceted across a 240 V, $0 c/s supply, Determine: {a) Total impedance, ¢b} Total eurrent {e) Voltage across each component (a) Power factor (©) Power consumption in (9) Frequency at which resonance will occur. Draw the complete vector diagram. Solution: top 92M 20a fa) R= 10082 4 Gy eB NX, = 2m fL = nx 50% 0.2 =62.39. ' be nov o Meo 0 oni Fig. 1.61 1 2a 50% 20% 107 = 1590. 86 Basics of Electrical E be, 10854 IRE 172.5¥, 1% =%e) 2 1057¥ Es2av + o s2T42v7 Fig. 1.62 Net reactance ¥ =. - = 100? +(-96.2)° = 1390. Ans. 240 b ct =o = L725 Ans. h total current J= 55 = L728A Ans {e) Voltage across resistance, EB, = 1.725 x Mi0= 172.5 V Voltage across induetance, By «1.725 x 62.8 = 108.5 ¥. Voltage across capacitance, Ec= 1.723 x 159=274.2¥, Ans. R100 (a) Power factor cas = 5 = Z 139 The power factor is leading as capacitive reactance in greater than the inductive reactance ia the circuit {e) Power consumption in the circuit = 0.72 leading Ans, Elcos Aernating Current Fiendamentals and Cinewit 87 = 240 x 1.725 x 0,72 = 298 watis, Ans. (0 Resonance will occur, when on 2a fL 39.8 cl. Ans, Qny2x 20x10 Example 33: A circuit consisting of resistance of 10 Q in series with an X) = 15 Qis connected in parallel with another circuit consisting of resistance of 12:2 and capacitive reuctance of 20 G combination is connected across a 230 V, $0 Hz supply. Find (a) Total current taken from supply (b) Power factor of circuit, Solutio (a) The given circuit is shown in Fig. 1.63. toa mwa 20, + 2a0v . Fig. 1.63 Branch I: Conductapee gy = 0.0307 2 88 Basics of Electrical E Susceptanee fay = Fr == R046 Branch I: Conductance gy = 4 = <2. = 00225 SZ S20 Since, branch 11 has capacitive susceptance, so itwill be assigned —ve sign 3 420 0.068 : Suscep Daw nce by = Combined eireuit Total condui 198 g = 0.0307 + 0.022 = 0.0527 B R68 —4.0461 = - 0.0093 Total conductance b = b, ~ b, ” Total admittance ¥= g* +5 = 0.0527" + 0.0093" = 0.0535 Q < Current taken from suppl I-EY-230K 00835-1234 Ans. (b) Power factor cos 9 = _ 0.0527 = 0.985 lagging Ans. psig 7028S Iagting Ans Example 34: [na parallel circuit, branch I consists of a resistance of 20. in series with an inductive reactance of [3 @ and branch Il has a perfect condenser of 50. reactance. The combination is connected across 200 V, 60 c/s supply. Calculate: {a) Current taken by each branch (b} Total current taken, (s) PVE. of the combination, Draw 9 Aernating Current Fiendamentals and Cinewit 83 Solution: 20 Ba teas oo + 200" 4 -Ba “ 200v » Fig. 1.04 Fig. 1.65 (a) Branch 1: a> 20 +15 = 250 200 ” 1 8A 25 = tan 8 ~ 36.9" lagging o> tan aging Branch I: +(-50" = 502. 200 * 30 4a, 4 L ' = tan Stan |e 0 = 90? leading. (b) Combined circuit: Te Resolvi ial current / is the vector sum of the two branch currents [, and fy in their active components). Toos =F, cos, + J; 08 = 8 cos 36.9° +4 cus 90° =8x08= 6.4.4, ho currents along 90 Basics of Electrical E Resolving the currents perpendicular 1 E (ic, im their rective components). Isin 6 =i, sin 9, ~ 1, sin 6, 4 sin 90-8 sin 36.9" 4-8 x 0.66 0.8 A. ” re oa + Cosy = {c) Power factor of the combination, Example 35: Solve Ex. 3 Solution: G0 715) 320) n = 1204 300) + 74180 = 200) (22=f5) (224 j5 _ (420% 22-+ 20x 5)+ J (420x $= 20% 22) 24+F 95. a = SO 1660 eas 4 3.26 509 fa) + (by Aernating Current Fiendamentals and Cinewit 91 Series-parallel eireuits: Consider a series parallel cirewit as shown in Fig, 1.66. In this case the parallel circuit is first reduced to its equivalent series circuit and then combined with the rest of the circuit, Re R, % ty gig + t 3 WG Ry 1.66 Example 36: A series parallel circuit is shown in Fig. 1.67. An alternating voltage of 200 V, $0 o's is applied across the terminals | and 3. Caleulate: {a) Symbolic exp: (b} Brach currents and total current {e) PE. of the circuit, (a) Voltages across the series branch and the parallel combination, {¢) Total power absorbed in the circuit ession of the total impedance of the cireuit Draw the complete vector diagram. zea ssa +e bot wa 92 Basics af Electrical E equivalent impedance of the parallel cireuit, ~ G- s+ 44+) “A G+ ABA) _ 4 THI T-fl T#J _ 175 = j25 175 725 Fe 50 =35 p05. Symbolic expression of the total impedance, Zz =(QS+715)+5 705) =6+s1. Aus, Teking the voltage as reference veetor 00+ FO. £2004 9 zy bal Total current £ 2006 —j 1) = 3204-54-04 RBA Ans. 54 and, o=tan | (- ) = tan! (-0.1665) = —9.5° Aernating Current Fiendamentals and Cinewit 93 Voltage across the series branch, E = F.2,= (324-54) 2S 471.5) = 89.14 735.1 8.1 >= tan! |] = san" 0,394 = 218° oon’ (24) Let the effective value af resultant current is 1 Instantaneous current = $ + § sin éoy = Pra SR+ ( 1=6.12amp Example 44a: [f the current in a 200) resistor is given by i = 4 + 3 sin cor 3 cox 3 car. Determine the pawer consumed bY the re 104 Basics of Electrical E Solutions Po Pat Pye Py = 4) 20+ 20x = (16 + 12.5 + 4.5) x 20 = 660 watt, 66 20 ample 45: A large coil of inductance | 405 H and resistance of 40 9 is connected in series with a capacitor of capacitance 20 uF. Calculate the frequency at which the circuit resonates. Ifa voltage of 100 V is applied to the circuit at resonant condition, calculate the current drawn from the supply and the voltage across the coil and the eepacitar, qualitY factor, band width, efietive value of current V5 = 5.7 Amp Solution R=40O, L= 1405 H, C= 20% 10° F l t AmVLC an 405 20x 107° resonant frequency = 20 Ha, Atresonance current fy = At resonance impedance Zy = R + JX, = JRP+ AX], = (40? 4264.87 = 2674.0 x, > Voltage across the coil al resonan 4 V), = Ip Zy= 2.5 X 267.8 = 669.5 volt = Capacitive reactance at resonance is Xp, Xe = = 20520 © Qa fC” 2m x 30x 20x10 => Voltage across the capacitor Ve, = Nelo 268.22 2.5 = 663 ¥ W 2 2mx 30x 1405 => Quality tuetor Qy~ Mob 2 LM 2 1405 6.6175 Ans. R R 40 => Band width = > Aernating Current Fiendamentals and Cinewit 4105 Example 46: A current of 120 J 30 flows through a circuit when the applied voltage is $ + j 2, determine (i) impedance (ii) power factor (iii) power consumed and reactive power. Solution: v= +s2W=828 21 ¥ 4=(120—/ S0}A = 130 2 ~22.62° A y ga5 Zi ‘i zo SAW op n63s 2 36.02°0 1 1302-22.62° “a Z~ 0.0635 2 (iy $= 36.62" lag. pl. = €08 0 = cos 36.62° = 0.803 lag Gi) Complex MA, 5 = Phasor voltage x conjugate of phaser current orp + jO° 8.25 2 14° x 130 2 22.62% © 10725 £ 36.627 Ht = 1072.5 feos 36.62° + j sim 36.62°) = (8608 +7 639.75)HA <+ Power consumed, P= 860.8 W Reactive power, @ = 639.75 VAR Example 47: Inan &~ L seriescircuit R= 10 and X= 8.66 @ if current in the circuit is (5 —j 10), find Gi) the applied voltage (ii) power factor and (iii) active power and reactive power. Solution: Z=R+JX,= (10 +866) Q= 13,23 £40.92 £5 (8-4 10) = 1118 2 -63.43° A a) Fe H2= U8 £6343? x 13.23 240.9" = 148 £22599 ¥ a“ ¥ = 148 Volts. ai @ — 63.43°~ 22.53°= 40.99 p.f. = cos @ = eos 40.9° = 4,756 lag. (ii) 5 = phasor voltage x conjugate of phasor current. or P+jQ= 148 2-22.53" x 11.18 2 63.43° = 1654.64 2 40.9° Fd, = (1250.66 +7 1083.36)04 “ Active power, P= 1250.66 W Reactive power Q = 1083.36 VAR Example 4%: An inductive eoil is connected in paralle! with a pure resistor of 30 and this parallel circuit is connected to a $0 Hz supply. The total current taken from the circuit is 8 A while the current in the resistor is 4A and that in inductive coil in 6 A, Calculate (i) Resistance and inductance of the coil (ii) Power factor of the circuit and (iii) Power taken by the circuit 106 Basics of Electrical E Solution: The second branch bas a pure resistance (Z, = 30 ) so that current J, (= 4A) will be in phase with the applied voltage. The first branch has an impedance of Z, and current (r* 6 A) through it wall la the applied s shawa in voltage by O) the line current (= SA) is the phasor sum of J, and f, the phasor diagram in Fig. 1.73, (i) Supply voltage, F = 1,2) = 4 30 = 120 Coil impedance, £, = F/I, = 120/6= 20 Referring to the phasor diagram in Fig, 1.72 we have, PaIF+H+2jl, 0090, Pe 2K Hoe 4 x C08 or, .25 and sin $= 0.968 Leda v 0A bison ¥ . 2 ’ er ¥, Sonz Fig. 1.73 + Coil resistance, R= Z, ¢03, = 20x 025 =52 Coil reactance, VW, = Z, sing, = 20 4 0,968 = 19.36 2 Coil reacwnee, b= #2 1936 goign. 2nf 2x50 Gi). Resolving the currents along x-axis (see Fig. 18.8) Joos = 1,41, €0s 4) 1410050, 446% 0.28 i 8 = 0.687 lag. Citi} Power consumed, P= FF eos @ = 120% 8 3 0.687 = 660 W. wt circuit PE, eos 6 = Example 49 An iron cored coil 4 connected to a 160 V, 54 Hz supply is found to take 9 current of $A and to dissipate a power of 200 W. When the same suppl¥ is connected lo coil B, the eurcent is 8 A and the power dissipated Aernating Current Fiendamentals and Cinewit 107 is 450 W, Calculate the current and power taken when the two coils are connected in series across the same 100 V, $0 Hz supply. Solutio Colla . Pym TG Ry 20 (5) Ry, Ry = = =82 B18 99 i, 3 Industive reactance Kye Vzi- Re 207-8 = 1833.0 Zypo Rye jApe Bes 18332 Col B Pee TRy 5 450 (8F Ry. Ry = 2 =7.032 fy 10 sa Inductive reactance = J12S- 703" = 10360 io ti Ky = 7.03 + 10.36 the two calls are canneeted in scries, the total Bm 2,4 Zy~ 845 18.334 7.03 Whi = 15.03 +) 28,69 = 32.388 262.382 Current taken by the combination V_ 109.20" Z 32.388 262.35° Power dissipated by the combination P= VI cos 0 = 100 x 3.087 cos 62.35° = 143.26 W on, P= PR= (3,087) = 18.03 = 143.26 W i = 3,087 £-62.35°A 108 Basics of Electrical E Example 50: Two circuits having the same numerical ohmic impedance are Joined in parallel, The power factor of one circuit is 0.8 and the other 0.6. What is the power factor of the combination? Solution: Let Z be the impedane 2, > ZL cas 08 = ZZ 36,87° = 240.8 +7 0.6) 2-2 Zcas'06=2253.1 Z (01.6 + 0.8) Since, the two impedances are connected in parallel, the equivalent impedance of the combination is given by eh circuit ate 290° Z 245° ZO L455} 198 The power f cos 0 = os 45° = 0,707 tor of the combination is Example 51: A voltage of 200 2 30° V is applied (o two cireuits connected in parallel. The currents in the respective branches are 20.2 60° A and 40 £30" A. Find the kWA and KW in each branch cireuit and in the main circuit, Solution Vs 200 4 30° V= 173,24) 100V 4 = 20.2 60° V= 1047 17.32.A 4740.2 -30° V= 34,64 209 Sy = HEY = (200.2 30°) (20.2 -60") = 4000.2 30 VA = 3464 ~/ 2000 4000 VA = 4RVA ~ real part of Vy = 3464 W = 3.464 KW. Sy = FL, = (200 2 30°) (40 2 430°) 8000 60° VA = 4000 + 7 6928 5, = 8000 VA = 2 BVA, P, = Re[¥T*] = 4000 W = 4 kW “ > Aernating Current Fiendamentals and Cinewit 109 Total current of the circuit FT) + Iya WW 471732 + 34.64 720 = 4446 —f 2.68 = 44.72 2 343° A S= VIF (200 230% (44.72 2 + 3.48) = 8944 2 33.43" VA ~ 7464 + 74927 P +j0 S= 8944 VA = 8.944 VA Ps 7464 W = 7.468 KW. Example 52: Three impedances (6 + /3) 2, (& — j6) Q and (8 * /L0) Q are connected in parallel. Calewlate the current in each branch when the total current is 20 A Solutio Let the tos rent f be taken as refer mee phasor. 122020 = 204704, 2-84 fS~ 781 239.82 2. 6 75 (6+ j3)(6= 73) js = 5-18 <9 o9g36—j 0.08196 § ors &-f6=10 2-36.87 2 i = 0.08 +/ 0.06 8, = 84 flO 128 2513490 8-10 48+ f10)(8— f10) 8? +107 = 0.04878 — j 0.06097 S f the circuit Y=¥,4¥y4 = 0.00836 ~ 7 0.08196 + 0.08 - j 0.06 + 804878 — 7 0.06097 Total admit: = 0.22714 —/ 0.08293 = 0.2418 £-20.06° 5 110 Basics of Electrical E Total circuit voltage vem=Lo 8 971 220068 Y 0.2418 2-20.06" VoL 82.712 20.06" 4 1812398" =10.592-1 qe 74° A= 9.967 —j 3,576 A, = 8271 £56.99" A= 4,513 +) 6.9304 _ #8271 220,062 2 1282-5134" 4 = 646 2-31.28" A= 5.52-j3.35 A Nthtk~20+j0-7 Example 83: A single phase circuit consists of three parallel branches, the admittance of the branches arc ¥,=0.44j06 FH = O14 j04 ¥, = 0.06 +j0.23 Determine the total admittance and impedance of the eireuit Solution: Since, the admittances are in parallel Yo¥y+ Yat = (04 + / 0.6} + (0.1 +j O42) * (0.06 +7 0.23) = (04 + 0.1 + 0.06) + j (0.6 + 0.42 +0.23) = 0.56 +) 1.25 = 1.369 £65.86 Impedance = 2 = = = ———___ = 0,73 £- 65,86 ns F 1.369.265.86 Z= 0.298 —j 0.666 Example 54: Four loads are connected across a 230 V, $0) Hz line: fa) lights 10 KVA at unity power factor, (b) a motor 4 kW at power factor 0.8 lagging Aernating Current Fiendamentals and Cinewit 114 {e) a rectifier 3.6 kW at power factor 0.6 lending (d) a capacitor 8 kVA Determine the total kV, the total kWAr, the total kVA, the overall power factor, and the total line current. Solution: By convention, Q, is taken positive and Q,. negntive. Load a cos }, = Ising, = 0 S,=S, £9, =, (6084, +f sin 0,) = 1041 +f O)= (10 +f 0) KVA 1 C05 9, O8, sing, = 0.6 P,=Zjsino,) = 6 (0.6 + j 0.8) = (3.6-J 4.8) AVA Laod ¢ 08 fe = 0.6, sin 9. = 0.8 P= §,<08 8, =S, 2-0, =8,(008 4,=J i 93 = 6(0.6- 0,8) =43.6-J4.8V VA Load d For a capacitor, the current leads the voltage by 90°. Therefire, the power factor cos @ is leading. cos 6, = cos 90° = 0 (leading) sin gy =sin 90° = 1 P,=S,coxdy Py= 8x0, Py= OW Sy > 8,2 - 90° = 8 2 90° = (vos 90° —f sin 90%) =8(0-fD) = 0-8 RVA Total complex power S=5,+5 =10 7 Sy O+44+/3+36-f4840-f8 412 Basics of Electrical = 17.6 -j 9.8 = 20.144 2 -29.1° KA S=P+jO-S26 S= 20.144 BVA P=176KW Q- 9.8 kVAr (leadin; Overall power factor cos 0 = cos (78. 1°) = 0.8737 (leading) Total current KVAX1O0D 2014451000 5 55 v 230 In the network shown in Fig, 1.74, determing 4a) the total current in each branch, (d) the overall e power, and (g) reactive voll-amperes Example $5: impedance, (b) the total current ¢c) th power factar, fe) volt-amperes, (£) a¢ ra 0015H 180 pF Fig. 1.74 Solution: (a) Branch 1 R= 7 QE, = 0015 Ny, = In fly = 2x 30x 0018= 4.719 2, = Ra JX, = 7494.71 = 8.437 233.98 Branch 2: Ry = 12 QC, = 180 pF = 180 108 F ‘ | = 17.68 2 In fC, In 5x 180 x 10° 12 -j 17.68 = 21.37 2—55.83°-0 EX Aernating Current Fiendamentals and Cinewit 113 Since, Z, and Z, are connected in parallel, their equivalent impedance Z, is given by 2, 2y\\ 2s 2, +2, — (8.437 £33.99) (21.37 £-55.83°) (7+ j4.7) 4 (12 — f1 7.68) — 180.3 £~21.9° _ 180.3 £ -21.9° = 1.97 BL-34P 1 = 7.839 2+ 124° = 7.656 +7 1680 Branch 3: Ry = $2, L,= 0.01 H X,, = In fly= 2x 50x 001-3160 Zy=Ry+jN,, = S+/3.14=5.9 232139 2 Since, Z, and Z, are connected in series, the total impedance of the circuit 2-2, +Z,- 5+ j3.14 + 7.656 +f 168 ~ 12.656 + 7 4.82 ~ 13.54 £20,85°0 éb) Let the supply voltage V be taken as reference phasor. Vo 230 £0 ¥ = 230470 V, By Ohm's law total circuit current is vo 23020" jets = 16.99 7-20.85" A Z 1354 220.85" — 7 6.086 fe} Voltage drop across Zy is Vy = 12, = (16.99 £~ 20.85%) (8.9 £32.13) = 100.24 2 11.28? = 98.3.4) 19.6 By KVL V= Pigs Vac Vyc= PV yy = 230 +70 ~ (98,3 47 19.6) - 131.7 -j 19.6 ~ 133.15 2 8.469 V 14 Basics of Electrical E Alternatively, 1, = 12, = (16.99 2 -20.85°) (7.839 2 124°) = 133.18 2-845°V = 131.73 -j 19357 V Since, Z, is the parallel combination of Z, and Z3 =P Fac 133.15 2-846" V By Ohm's law v © dyn th = URNS EBA as og 242.399 A BABT 233.93 ir, = 11.65 =f 10644 gy= by = BRIS 2-846" 21.37 2-55.83" = 6.23 247379 A= 4 Check fy + Jy = 1165 710.64 + 4.2 (ay Ta pass pas8~l 87-7.6.06.A=1 current 1516.99 £2085" & ample 56: A coil of resistance $0 Q and inductance 0.318 [1 is connected n parallel with a circuit comprising a 75 (2 resistor in series with a 159 a capacitor. The circuit is coanected to 240 V, 30 Hz supply. Calculate, (ip Supply current (ii) Phase angle between supply current and applied voltage. Find also the sesistanccand reactance of series circuit which will take same current at the same p-f, as the parallel eineuit Solution: Xp = 2 nf, 2 ex 50 x 0.318-= 100.0 Z, = (R+Xj7 = 50? +100? = 1120 cos! Ry/Z, » cos” S012 » 63.58 lag, [= VIZ, = MONI2= 2.15 A Taps V’ bY 63.5° 10° Aernating Current Fiendamentals and Cinewit 415 VIZ, = 240/77, = 3.09 lead by 15° Fig. 1.78 Referring to the phasor diagram shown in Fig, 1.75 Total X-comp. =f, cos, + 4; 608 03 = 2.18 cos 63.5% + 3.09 cos 18° = 3.94 A. Total ¥-comp =~ 4, sin 9) + Asin Gs == 215 sin 63.5 + 3.09 sin 15° == 113. Gi} Supply current, = f(394)* + (= 13) = 4.14 (ii) Phase angle. 6 = tan =1.13/3.94 = ~16° Since, voltage in the reference phasor, negative angle means that supply current lags the applied voltage, i.c., circuit is inductive. = 140/41 = 5852 = 58.5 cos (16°) = $6.20 Circuit impedance, = 2= 1" Circuit resistance, R= Z cos Circuit reactance, X, = sin Q = $8.5 sin (-16°) = 16.12 2 Thus, the parallel circuit is equivalent to 56.2. resistor in series with 16.12 92 inductive reactance, Example 57: A voltage of 230 230° V is applied to two circuits connected in parallel, The current in the branches are 20 260° A and 40 2- 30° A, Find (i) The total impedance of the circuit (ii) Power taken 20 260 Solution: J, 20 (cos 60 + j sin 60°) = (10 +f 17.3) 4 1, = 40 £-30° 4 = 40 (cos 30° ~j sin 30°) = (34.6 — 720) 4 = (446 23.464 Ty 44 116 Basics of Electrical E i 11,20 £60". LAO 230° 230 0 Fig. 1.76 Vv _ 230230" 3.14 £33.46 2 1) 447 273.46 = 230 x 44.7 x cos 33.465 = 8577 W, Example 58: A parallel! circuit consists of a 2.5 uF capacitor and a coil whose resistance and inductance are 15 © and 260 mil, respectively. Determine (i) the resonant frequency (ii) Q-factor of the cireuit at resonance ii) dynamic impedance of the circuit. Solution: (i) Resonant frequency, 1 10° 1sy In 0.260% 25 (0.26 = 197 Hz Qn _ IR 197 x 0.260 R 15 0.260 25%10 ° x15 (ii) Q-factor (iy = 6933.2 CR Example $9: A square coil of 10 em side and 100 turns is rotated at a uniform speed of 1900 revolution per mintite, about an axis at right angles to @ uniform magnetic field of 0.5 Whim?, Calculate the instantaneous value of the induced electromotive force. When the plane of the coil in (i) at right angles to the field (i) in the plane of the field. (Electromagnetic Theory, AMIE, See B, 1992) Aernating Current Fiendamentals and Cinewit 417 Solution: Let the magnetie field Lie im the vertical plane and the coil in the horizontal plane. Also let the angle @ be measured from V-axis, n{NB,, A volt insta Maximum value of the induced emf. value of the induced em.£ ¢ > £,, sin Q. Nows= 10/60 O/S) eps. N= 100, 2, — 0.5 Whi (i) In this case, 8 = 90° Gi} Here, @=90°, ne im Sin 90° = E., Substituting the value (given, we get, = 2m x (503) x 100 x 0.5 x 10°? e=523¥, Ans. ‘e cous, =10 Example 60: The maximum values of the alternating voltage and current arc 400 V and 20 A, respectively in a circuit connected to $0 Hz. supply and these quantities are sinusoidal. The instantancous valucs of the voltage and current ate 283 V and 10 A, respectively ats = 0 both increasing positively. (i) Write down the expression for voltage and current at time (ii) Determine the power consumed in the eiteuit, (Elect, Engg, Pune Univ.) Solution: (i) In general, the expression for an AC voltage is 1”= F., sin (we + 0) where its the phase difference with respect to the point where t= 0. Now, P= 283; Mn = 400 F, substituting ¢= 0 in the above equ: 283 = 400 (sinw x04) + sing= 283400 0.707, O= 45° or R/4 radians, we get Hence, general expression for voltage is F = 400 (sin 2m x SO 1+ m4) Similarly, atr= 0 10= 20 sin Gv XO +9) * sin9=0.3 + = 30° oF w6 radians, Hace, the general expression for the current is f= 20 (sin 100 mt + 30°} = 20 sin (100 m+ 1/6), (ii) P= FT cos @ where Vand J are RMS values and @ is the phase difference between the voltage and current “al 2 = 400/-J2 5 1 2 77) Now, ve 0 = 15° (soe 118 Basics of Electrical E Fig. 4.77 n P= (400) V2 ) x (20 V2 J x eos 15° P= 3864.0. EXERCISES 1. What is meant by an alternating quantity? Explain how a sine wave is produced. 2. De! 3. What is understood by “phase difference” between two alternating quantities? Explain the (erm lagging current and leading current with the aid of suitable curves, 4, Define RMS value of an alternating current Derive RMS value in ease of a: (a) Sinusoidal wave (b) Rectangular wave cle, periodic time and frequency. (e) Trianguler wave (d) Semicireular wave (e) Trapezoidal wave (g) Stepped wave. 5. Define average value of an alternating eurrent, Derive average value in case of (a) Sinusoidal w: (b) Rectangular wove (c) Triangular wave (a) Semicireutar wave (e) Trapezoidal wave (g) Stepped wave. Also define form factor and find its value in case of all the above waves: Aernating Current Fiendamentals and Cinewit 119 6. Define peak or crest factor and state its practical utility. 7. Determine average value, effeetive value and form factor of a triangular wave whose half wave is suppressed in each cvele , tn 8, Two waves represented bY ¢) = 3 sin wi. and e, = 4 sin (4) are ting in a cireuit, Find an expression of their resultant and check the result by a graphical construction. Also find the peak and RMS values (N37 sin (wr, 0.608); V37 , 4.3) 9. An alternating current is given in amperes bY the expression, J = 50 sin 44 04 Find (a) frequency. (b) .w in radians per s (c} maximum value of the current (d) effective value of the current [Ca) 70 c#s, (b) 440, (¢) 50.A ond (4) 35.35 A] 10, Show thot in a purely resistive circuit, the current and voltage wave are in phase. Draw curves ofthe applied voltage, current and power. Prove thal the average power in the circuit is equal to the product of effective values of voltage and current. IL, Show that in a purely inductive circuit, current lags the applied voltage by 90°, Prove that the average power absorbed in such a eireuit is zero, 12, Show that in a purely capacitive circuit, current leads the applied voltage by 90°, Show that the power wave fluctuates with double the frequeac¥ and that the average power absorbed is zero in such a circuit 13, What is meant by: (a) inductive reactance. ‘ond () capacitive reactance (c) impedance SpecifY the unit of each one of them, Also express them in their s¥mbolic form. 14. Calculate the reactance of a condenser of capacitance 100 uF and inducing coil of 20 mA at (a) 25 c/s. (b) 50 crs and (¢) 100 c/s [(a} 63.6 Q, 63.14. (b) 31.8.Q, 6.28 Q (c) 15.9 Q, 12.56 Q] 15. A set of 200 V lamps take 2 4, Find inductance of a choke coil to be connected in series with them so that they may work satisfactorily ona 250 V, 50 eyele supply mains. Draw vector diagram (0.238 H] 120 16, 19, Basics of Blectrical Engineering A current of 10 A flows in a circuit lagging behind the applied voltage of 100-V by 30°. Determine the resistance, reactance and impedance of ihe cireuit. (8.662, 5 0, & 100) A non-inductive resistance in series with an ideal inductor is conneeted to a 125 V, 50 cis supply. The current 2.2 4 and the power loss in the resistor is 96.8 W calculote the resistance and the induetanee, Draw the vector diagram for the eircuit 1 the circu [20.2, 60 nF] A circuit consists of resistance of 20 ©, an induct are of 0.1 H and a capacitance of 250 [LF all connected in series. The combinat connected across 200 V, 30 ¢/s mains, Calculate (a) Current (b) p.f. (¢) power, (d) voltages across the coil and condenses. Draw vector diagram. [(a) 7.32 A, (bj 0.73 lagging, (c) 1070, (d) 273 ¥ & 93.2 Vy] State the condition of resonance in a series #i-C circuit, Define resonamee frequencY and express it in terms of & and C. An alternating vollage of 1 volt having a frequency of 100 kilo-cycles is, applied across the combin: coil has a resistance of 250 G2 and inductance of 12.5 wH¥. Determine the capacitance of a condenser which will produce resonance. Find also the power dissipated in the circuit and the voltage across the coil, Draw the vector diagram. (20.6 x 107 wr, 0.005 W. 31.4) Define conductance, susceptaace and admittance, and express them in terms of &, Yand Z. A coil of resistance 15 @ and inductance 0.05 # is connected in parallel with a non-inductance resister of 20 9 Find (a) the current in each branch circuit (b) the total current supplied and (¢) the phase angle of combisation when a voltage of 200 V at 50 cls ix applied. Draw the vector diagram. ((a) 9.2 A, 10 A (b) 17.6 A, (e) 22° lagging] ‘Two circuits, the impedances of whieh are given by Z,~ 10-4715 Qand Z, = 6 —) 8 O are connected in parallel, If the total current supplied is 15 Acwhat is the power taken bY each branch, Draw wector diagram, (737 W, 1400 W) A small | = phase 240 Y induction motor is tested in parallel with a 160.2 resistor, The motor takes 2.0 A. and the total current is 3 A. Find the power and power factor of (a) the whole circuit (b) the motor, Draw veotor diagram, [{a) 580 W, 0.807 lag (b) 220 W, 0.46 lag.) Find the total current taken and the power factor of the eircuit Fig. 1.178. Draw also the vector diagram. a is 1 of a coil and a condenser in series. The Aernating Current Fiendamentals and Cinewit 4124 son 04H Poe 200K TS 2m 04H See 4 20015 . 50. Fig. 1.78 [3.85 .4, 0.95 lagging] 25, Explain the phenomenon of “Parallel Resonance” A coil of resistance 15.2 and inductance of 0.5 # is connected in series h a condenser. On applying a sinusoidal voltage the current is maximum when the frequency is $0 eis, A second condenser is connected in parallel with the circuit. What capacitance it must have so that the combination acts like # non-inductive citeuit at 100 c/s? Calculate also the total current supplied in each case if the applied voltage is 240 V. [6.74 UF, 16.A & 0.0648 A] tive, equivalent or dynamie impedance of a rejecter circuit reuit at resonance afler making pract 26, Define effec Find the current in # parallel assumptions. 27. Define Q-factor and determine its value in’ ant circuit (a) Series rese (b) Parallel esonant circuit Magnetic Circuits 2.4 MAGNETIC CIRCUIT The path followed by a magnet ne of force is called magnetic circuit, line of force follows a closed path ic., it cames back to the originating point. A ple mag shown in shaped specimen of iron wound with a suitable aumberof tums of insulated wire. When a current flows through the coil, lines of force are produced in the specimen. In this ease. the magnetie circuits are circular paths of lines of force as shown by dotied Hines: very Fig. 24 Let N = Number of turns wound over the ring J = Current in amperes 1+ Length of the mean path of lines of force. Magnetic Circuits 123 Field intensity Hf at the centre of the solenoid is given by, nM 7 If is the permeability of the ring material, then flux density B in the ring is given by, ME : B= WH Hol weblmet” If is the area of eross-segtion of the irom ring in meter, then flux produced is given by Here, Mis called magactomotive force of magnetic circuit, Mug HA is called reluctance. Mmm Fygt,A Reluctance MME Thus, on (2.2 Reluctance This is sometimes called "Ohm's faw” of magnetic circuit. Itresembles the expression for electric circuit, AE. sistance Current f= (2.3) 2.2 CIRCUITAL LAWS There are two cireuital laws of the magnetic field. First Iaw—(Amupere’s law) states that the total magnetomotive force acting round a closed magnetic cireuit is equal ro the algebr enclosed by the path. So, MMB = ZF (2.4) Where, X= M/in M.K.S. rationalised system sum of currents 0.4m M Gilberts in c.g.s. system, Second law—(Faraday’s laws of electromagnetic indu cmf induced aroun a closed path is equal to the negative rate vf chal magnetic flux linkages. n) states that the of tbe 124 Basic of Electrical E dy So, ee NTP #10" volts (2.5) where, N= number of turns of the coil o interlinking flux in lines 2.3. DEFINITIONS: 2.3.1 Magnetic Flux (0) It is defined as the wtal number of lines of force emitted from a source, Incas. s¥stem, its unit is Maxwell or lines and in M.K.S. is Weber. (1 weber = 10* lines), 2.3.2 Magnetomotive Force (M.MF.) It is the force which drives the magnetic flux through a magnetic circuit It is also defined as the work done in carr¥ing a unit magnetic pole once round a closed path of lines of farce. tv 6.g.8. sYstem, M.MLF. = OAR NI gilberts (2.6) In M.K.S. rationalized system M.MLP. ~ AY Amp. Turns. In M.K.S. unrationalized s¥stem M.MLF. =r Ampere Tums. (28) Fray he expressed at A. N may be expressed us 7. Then, N= AT ie. Amp. Tums, (2.9) 2.3.3 Reluctance (A) I he obstruction offered by the magnetic cireuit to the flow of magnetic flux, It is analogous to resistance in an electric circuit i zg ie (2.10) Incws, system: fis in em, ais incm?, andp =p, I, f= tem = 1 in vacuum in e. a= lem? Thus, the unit reluctance is the reluctance of 4 centimetre cuts Inc.g.s. sYstem no name is given to this unit. vacuum, Magnetic Cinewits 125 Inks. system: Fis in metre, a is in metres? and jt = bp tL, The value of fy = 4m % 107 in nam, unrationalized and the unit of reluctance is he s. units and 107 iam. 2.3.4 Permeance (P) tis the reciprocal of reluctance and is analogous 1 conductance in an electric circuit. Powe A211) Permeance may be defined as the propert¥ with which magnetic flux ean flow in a magnetic circuit In m.k.s. system, its unit is henry, 2.3.5 Reluctivity The reciprocal of permeability is-called rele iY and is usually denoted by S ved (2.12) i Reluctance = — = % ua The reluctivity corresponds to resistivily in an electri cireuil 2.4 SIMILARITY OF MAGNETIC & ELECTRIC CIRCUITS * unr? EME ) Magnetic creult (b) Electric ireut Fig, 22 126 Basic of Electrical E Magnetic Circwit Electric Circuit (i) Flus Corresponds to Current (ii) MAME » EMF (iii) Reluctance ” Resistance (iv) Permeance . Conductance 4) Flux density . Current density (lines!sq. em’) . (Amps/em*) (vi) Permeability (qr) . Conductivity (a) (vii) Reluctivity tv) . Resistivity (p) 2.5 DISSIMILARITY BETWEEN MAGNETIC & ELECTRIC CIRCUITS ()) Flux—No flow of flux in @ magnetic | Curvent—It actually flows in an electric Gircult rout (i) Permeanitiy—(a) It depends upon | Conductivity—(a) It does not depend the fotal fx. (b) It does not vary too | upon the current strength. (b) It varies much and so no material could be | within vary large limits and so materials Said 65 insulator lo magnetic lux. | are termed a5 canductors or insulators. (i) Energy—it is expanded in | Snergy—it is expended so long as the establishing flux but not for | current flows. maintaining i 2,6 RELUCTANCES IN SERIES A composite magnetic circuit is said 10 have reluetanees in series, if the same flux passes through the different branches. pis. a4, and a be the cros Ha My and |, be the permeabilities of the limbs 4, # and C, respectively as shown in Fig. 23. IER, Then, setional areas and Rj, and R, are the reluctances of the three limbs, and, Magnetic Cinewits 427 Fig. 23. IPA, My and Af, be the M.M.Fs of the three branches and 9 be the flux passing through them, then from eqn. (2-4) M,= OR, (2.13) My OR, (2.14) Mo=O.R=o (2.18) ‘Talal M.M.F. = AL, * aM, * M, (2.16) If the equivalent reluctance is R, then Total MNLF = 0.8 vo) Comparing eqns. (2.17) and (2.18) GR =O. + OR, + OR, or, R=R,*Ry* RB, (2.18) = + (2.19) Bede Myth Bed, Thus, when the reluctauces are eonaccted in series, the equivalent reluctance is the sum of the reluctances of individual branches. 128 Basic of Electrical E - Total MN. + Fo. 24 So the circuit of Fig. 2.3, may be represented by an equivalent circuit of Fig. 24. So the total flux is given by MMF. 23 _—— 2.20; H+ kh, +R, 2.20) _ MME. R. Inc.g.s. s¥stem, flux will be An NI oF OAR NE vu(2.21) = Ma 2.7 OHM'S LAW OF MAGNETIC CIRCUIT 2.7.1 In €.G.S. System MME = 0.dx NF gilbent where, = No. of turns, 1 = Current in amps a-t ja where, /~ length in em, a= area in em’, B= Hott Hl = | = Absolute permeability af free space i Ss. unit, H, = Relative permeability of the material, Magnetic Circuits 129 Thus, H=H, in eg. system, 6 = flux in fines, he = Odnat Upe Ni= AT = Ampere Tums. amar “ on H, 7 te Since So, ar= OAR La = 0.796 FL (2.22) Thus, ampere-tamns required ~ 0.796 x Magnetising foree x length of magnetic path. 2.7.2 In M.K.S. Rationalised System M.MLE, ~ Né ampere-turns o-(2.23) Where, N= No. of turns F Current in ampere. = Len. ha 2.8 RELUCTANCES IN PARALLEL When the flux established in a single coil passes through several parallel branches of magnetic circuit, the reluctances are said to be in parallel, In a simple magnetic circuit of Fig. 2.5(a}, the flax @ divides into two parallel branches, so 0-940, (2.24) Let M = M.MLFP. of the coil on central limb between the points 4 and B. Ry = Reluctance of the « Ry= Re R= Equivalent reluctance of both the paths, then jance of the eincuil 130 Basic of Electrical E 6 . ~~ 1 . = = . ME . 8 ta) Fla. 28 p= MME _ Mt R R oe Re Substituting the value of ), , and 6 in equation, (2.25) MoM | M l or, ~4 2.26 x (2.26) Hence, in general if R,, Ro, R equivalent reluctance is given bY. Thus, reluctances in parallel may be represented by an equivalent elet circuit shown in Fig. 2.S(b) If P is the total permeance of the circuit and Py, Ps, Py, ... be the permeances of the parallel branches, so that Magnetic Circuits 131 Pe ‘ P= Then, PoP APS (2.28) Hen permeanee . the permeances in parallel are added up ta obtain the total 2.9 ELECTRIC CKT CORRESPONDS TO MAGNETIC CIRCUIT (SERIES AND PARALLEL) 4 Equivalontectre CKT Fie. 26 Ry = reluctance fir path (1) to (3) = fy y/ity HA reluctance for path (1) > (2) (4) Ry = Feluctance for iron path (1) + (2) + (3) R= reluctance for air gap Net minf required to establish a Mux 0 is Ami) ~ 6fnet reluctance] {Wan} [ eed 9] Rk + RFR FR, 132 any tase peng) perey paeyge pege sus tye ‘uuyoae woe quscynos quigypoa quojpooqe quojnonge ued anode aeduede sedwede queuing Yor yon syonqe yonge (Apo gna an oUI/SyOA eaeUayan wonnange woywange Asus prey OURO awornos squoyn09 uy auwornoace a awojncode a (a) xy 201s Ave Anew Auayge Auayae (7) eauepnput enauiiN oun touysuin-due (worag)6) posse | (4) Asuaru ploy ogoutleny (ung suurg-dase | (ane ap) sunp-owe ny swn-owe in 40) Boa 010) onnoWOFOUBENY sutbiufsieaem eabotujsiogam aul "bsys0u (jwaysou) sens: eotouBony senam Jagan ‘uy 20 yee u mUTEN pasyeuoney possevonexn, speuBewonso(g waeKs 'SHN "S99 Ios pue ainuenD en [eOUoNY PLE oNeUBeYy LooyO LONE 1-2 O1GeL Magnetic Circuits 133 Example 1: A magnetic CKT shown in fig having a eross-section side of square cross-section of 3 cm side, Each air gap is 2 mm wide. Each of the coil is wound with 1000 turns and exciting current is 1.0 A. The relative permeability of part | and part B may be taken as 1000 and 1200, respectively. Find (i) Reluctance of part 4 and B (ii} Reluctance of two air gaps Gi) Total reluctance of the complete circuit (iv) MMF () Total Mux tren (vi) Flux density : : Parca — — hrocm Pars ba Fig. 27 Solution: i Reluctance Ry = ——— Holt (i) Foe part A . aon . Taam . iran . Fig. 28 hf, ~ mean length of core f= 20-(1S+15)= IT em 1-017 m My =4nx 107 #, = 1000, 4 = Area of cross-section = 3 x3 x 10m? 17x 107 = = 15,08 10 ATs 4x10" x 1000 9x10 134 Basic of Electrical E For part B Fig. 29 length Jy = mean length of core AH+BC+CD = (10 1.3) + (20-3) + (10-13) HIT +85 3dem dy 34x10 Hof, Aree 107 ¢ 12005 9 LO = 25.04» Lo! AT/wb ii} Rel jctanice for air gap R= 1 ag Me™ Hor air gop length of mean path for airgap mm =4x 107 metre Area of air gap through which flux passes = 9 x 10 4m? . 4x 107 amxlo’ x9 1074 = 353.5 x1 c circuit corresponds t magnetic CRT is Fig. 2.49 Magnetic Circuits 135 Total reluctance y= Rig * Rig Ry Ry = (15.03 + 25.04-+ R,= 393.57 x 10 ATIwb Exciting current is 1.0 A and number of turns is N= 1000, mmf = M/= 1000 x 1.0 = 1000 AT mmf 1000 Total fli (9) Total fax @ = Reiuctance ~ 39387 x10" 5.08 x 10-4 weber. 8.0810 * (vi) Flu vi) Flux x? 0.564 whim’. Example 2; A steel cing of 25 cm mean diameter and of circular section 3 em in diameter has an ai gap of | i is wound uniformly with 700 tums af wire carr¥ing @ current of 2 A. Caleulate (i) mmf (ii) flux density of airgap (iii) magnetic flux (iv) reluctance (v) relative permeability of steel ring (vi) Reluctance of stecl assume mmf taken bY irom path is 35% of total mnt. am lei Solution: ER, count mint 1 +R) Fig. 244 Fia. 242 Mean diameter = 25 em Number of turns N= 700 Current in a coil F= 2A, (i) moms NT = 700% 2= 1400 AT 136 Basic of Electrical E (ii) reluctance of air gap is Ry = Bod 1S mm = 1,5 x 107 metre, mmf across air gap = OR, 10> BLAS tmmi, = 8,4, R, (Potal mmf — 35% of total mm (1400-035 1400) = Bl ity 2910 dx 10? x O10 ty 13x10 8, 0.762 whine (Gil) magnetic flux = BA Fig. 2.13 Diameter of circular section ring = 3 cm Arca of cross-section = 277 -»(3) 7.02 «104m? flux $= 0.762 x 7.07 «10 9 = 0.538 m Weber (iv) Total mmf= 1400 AT total flux = 0.538 x 10? weber 1400 dasexto? 2° 10° ATAwb: Magnetic Circuits 137 (v) Let relative permeability of steel ring is, fy R- Malta 15x 107) Ls x10" an 107 x 7.07x 10% 88.79 A, = 0.01689 x 10% = 1.69 x 10! Reluctance of steel care = R,— RK, = (2.6 = 1.69) x 10° 0.91 x 10" ATANb Esample 3; Magnetic CKT shown has cast steel core with dimension. Mean length from 4 to B through outer limb = 0.5 m Mean length from to B through central Himb = 0.2 m % 1 ‘6 -Vxton 0.02 om oalemi "0.028 em Fig. 244 In the magneticCKT shown it is cequired to establish a flux of 0.75 msvb in the air gap of the central limb, Determine the mmfof the exciting coil if For the core material (a) {l,= 2 (b) tt, = S000 {a) [, ~ ethore are no mint drop in magnetic core. Nt Fig, 215 138 Basic of Electrical E Lp. 0.02510" a ‘ 5 1.99% 10" hata” arx1o xt 10" = 7 = 1592 x10" Rx 10 x1 10 re = 0.02 «10 = = 0.796 x 108 IRxIO x(x 2) 10" Ma OLR, + CR, 8) Mi = 0.75 x 10°%(0.796 + 844) x 10° = 1260 AT This means that the reluctance of magnetic core must be taken into consideration, Fig. 246 L 05 8 pga dx OT x $000 (1x 1) x 107 t= 0.796 x 10° 02 Fe Pay = S96 10° 4xx 107 x S000 (2x1x 10%) UR # RVR, + Bg Ry + Ra Ae 10% Magnetic Cirewids mmf= oR, (Ved R,) 0.75 x 107 x «10° [mmf = 1680 AT) 139 ample 4: An iron ring 10 cm means diameter is made of round iron rod 1.5 cm in diameter of relative permeability 900 and has an air gap 5 amm in length. It has a winding of 400 turns. [f the current through the winding amps. Determine (#) MMF (b) total reluctance of the eireuit (¢) flux Fing (4) flux density in the ring. Negleet leaka, Solution: The magnetic eircuit is drawa in Fig. 2.17 below. w=800 0.0m Fig. 247 Length of air gap—J, = 0.5 cm. Length of iron path—f, = (x 10 0.5} = 30.9 em, Rx IS? Area of cross-section, a= = LTT om’ a AnNE &) MME = gilberts mx 400% 34 = oo gilberts (b} Total reluctance = —— aii; 34 the 140 Basic of Electrical E _ 309 177x900 = 0.0194 + 0.272 as L771 a. 4 171 ~~ $870 maxwells fe) Om - ‘ Reluctance 0.2914 & _ S87 = 22 SS 3920 gnase Ans, W #-Te Te 8 Example 5: In Fig. a ring ef composite material is shown. ‘The length af the i, cast steeh and ait gap are magnetic path and cross-sectional areas in ir 100 em, 200 cm and | cm and 20 em*, 10 em* and 26 cm’, respectively. The length of path in icom is divided into two equal halves of 506m each, If the relative permeability af iron and cast steel are 300 and 900, respectively. find the current through a coil of 170 tuins to produce a useful flux of 9000 Fines in the air gap. Take leakage coefficient as 1.2, so en 10cm. Tom 170ums Cast steel — Fig. 218 Solution: (In R.MLK.S. Units) MMF for air gap (Mg.) M.A for air gap(Mg.) Reluctance Flux in air gap = Mg, = Flux in gap * Reluctance of air gap Ig ___ length of the path in gap in m Hat, py, area of gapinn Reluctance = Magnetic Circuits 141 ixtot am x 107 x 1x 20% 10 1x10 o Mg = 9000 x 10-8 3 ———>—____ Axx x 20x10" = 358 Amp-turns. MME for Iron path (af Mi = Total flux in iron x Reluctance of iron paths. Total flux in iron = Leakage coefficient x Flux in gap = 1,2 x 9000 x 10° = 10800 x 10°* wb, ‘ 003 10? * Mi = 10800 x 10% 100%) ; 4nx 10" * 300% 20x 10 = 143 Amp-turns. MMEF for cast steel path (Mec). Mc = Total flux in cast steel x Reluctance of steel paths. 200% 10"? = 10800 x 10 ¥ x FRx TO” x 00% 10510" 91 Amp-turns. e. Toral MM required -Mg + Mi* Me. = 358+ 143 + 191692 AT, Total M.M.F. required Hence, current through the coil = No.of tums in coil ~ 2 4.06.Amp Ans. 170 Aliter (in e.g.s, system) Amp-turns for the gap. From eqn. (5.23) ATg = 0.796 He. te. Flus in gap i, area of gap where, Mg 142 Basic of Electrical E _ 9000 1x20 = 450 lg= te “ AT = 0.796 4 450 1 358 Amp-turns. Amp-tarns for e iron, ATI = 0.796 Hi. jux in iron aj = Flex in iron _ pox area of iron Flux in iron = Leakage coefficient x Flux in gop. 1.2 «9000 = 10800 lines. n= 300 1osoe 300 20 So {Fi = 0.796 x 1.8 x 100 143 Amp-turns. urns for cast steel, ATe = 0.796 Hele _ Flux in cast steel He Hu, _ 10800 | 5 900 x 10 ‘s ATe = 0.796% 1.2 «200 = 191 Amp-turns, So, total ampere-turns required “Ag © ATi + ATe = 358 + 143 + 191 = 692 Hence, current through the coil, =. Total amper ms required No.of tumsin theeail 692 179 PO AmpS Ans. Magnetic Circuits 143 I 2 3. 6 EXERCISE Explain the terms: 4) MME, ii) Flux (iv) Reluctance (Permeability (vi) Perme: (vii) Fringing (viii) Coereivity (ix) Retentivity Find the having a relative permeability 1, area of cross-section is. A ation between MMF, reluctance and flux for a circular ring What are the similarities and disimilarities of magnetic circuits and electrical circuits? Drow a magnetization curve and detine the hysterisis and eddy current losses. Explain the effect of AC excitation on magnetic circuits, (Hin Transformers and ac machines are excited from ac source) AOS m long wire moves at right angles to its Tength al 40 m/s in uniform magnetic field of 1 Whim’, Calculate the emf induced in the conductor when the direction of motian is (i) Perpendicular to field {ii) inclined at 30* ta the direction of field, [Ans: 20 ¥, 10 V] An iroa ring has a mean circumferential length of 60cm with an air gap of L mm and a uniform winding of 300 turns, when a current of LA flows through the cail, find the flux density. The relative permeability of iron is 300. [Ans: 0.1256 T] For the AC excited magnetic circuit given betow, calculate the excitation current and induced emf of the coil to produce a core flax of (0.6 sin 314.) mw, . 356m . Fla, 2.19 148 9, 10, Basic of Electrical Engineering A coil of insulated wire of $00 turns and of resistanee 422 is closely wound on iron ring. The ring has a mean diameter of 0.25 m and a uniform eross-sectional arca of 700 mm’, Calculate the total flux in the ring when a DC supply of 6 V is applied to the ends of the winding, Assume relative permeability of $50, [UPT.U, 2002] [Ans: 0.462 mWb) ‘An iroa ring of mean length 50 em and relative permeability 300 hasan air gap of | mm. Ifthe ting is provided with a winding of 200 turns and a current of | A is allowed to flow through. Find the flux density across the air gap. (U.P.TLU. June-2001] [Ans, 0.0942 T] ‘A coil of 1,000 turas is wound on a laminated core of steel having a cross-section of 5 cm’, The core has an air gap of 2 mm cut at right angle. What value of current is required to have an air gap flux density 010.5 T? Permeability of steel may be taken as infinity, Determine the coil inductance (2 = we), [U.P.T.U. 2003-2004] {Ans, 0.314 Ht] D.C. Network Analysis 3.4 INTRODUCTION In this chapter we analyse the linear network for Direct-voltage and Dircet- current For analysing these networks, we introduce two different laws KVL and different theorems, 3.2 CHARGE The basic unit of charge is the charge of the electron, when electrons are removed from a substance. that substance becomes positively charged A substance with an excess of electron is negatively ehanged The electron has a charge of 1.6021 x 10" caulomb and represented by @ org 3.3 ELECTRIC CURRENT The phenomendn of transferting charge from one point in a cirewit to anather is described by the term electric current. It may be defined as the rare of flow electric charge across a cross-sectional baundary. A random motion of electron, in. metal does not constitute a current unless there isa net transfer of charge with time = 4 at 3.4 AMPERES Ifthe charge g is given in coufombs and the time is in sce, then the current is measured in ampere, 146 Basics of Electrical E | ampere corresponds to ihe motion of = 6.24 x 10'* electrons 16x10" past any cross-section of path in one second, 3.5 VOLTAGE Once the battery circuit is closed by an external connection, the chemical ey (energy io chemical reaction) is expended as work. The enerey per unit i] A voltage can exist between a pair of electrical terminals whether a current is Mowing ar not. (According ta principle of conservation of energy, the em that is expended im foreing charge through the clement must sppe somewhere) else charge or work per unit charge is given the name voltage ( 3.6 VOLTAGE SOURCE Voltage source is ussumed to deliver energy with a specified terminal voltage, af), and may or may not be dependent of the current from the source. Voltage source is said to be ideal ifthe voltage docs nai depend on current ° ‘ va v o © “1 Symbol for ters ‘Symootfor ime (va graph for varyingvottage source} invariant voltage source deal vottage source so loa ~ practical Practical source 3.7 CURRENT SOURCE The current source is assumed to deliver energy wilh with a specified current through the temiinals s(t) and may or may net be dependent on terminal vollage. D.C. Network Analysis A current source is said to be ideal if it is independent of voltage Transistors, vacuum tubes and photo clectric cells make use of current source in their model representation Va mo ik ity ut eal source (doo! Practical curont sourca 3.8 POWER Power is the rate at which enemy is expended, and shall represent it by P or P. If one joule of energy is expended in transferring one coulomb of charge through the device, then the rate of eneruy expenditure in transferring one coulomb of charge per second through the device is one watt This absorbed power must be proportional bath to the number of coulomibs transferred per second or current and to the energy needed to transfer one coulomb through the clement or volt Joule. Coulomb Previn SEE © souteisee (watt) Coulomb “~~ Seo jve current catering at a ive terminal, then energy is being supplied to the element, Otherwise, the ¥ to some external ment is delivering ener device, ql vols & Element v (ay v Al v 8 e -i (a) PowerP =vilsabsorbes (0) P= (u) (P= EN) by thewlement is absarbed by alernert is absorbed by element (Tormpal A's ve tbe wrt ‘or wits Golarod by element letermirsl 8) = So, if one terminal of the element is v volts positive with respect to other terminal and ifa current is entering the clement through the +ve terminal, then the power? = vi is being delivered to the element or absorbed by the element. 3.9 SOME BASIC DEFINITIONS Some desirable definitions hetore going to further discussion. 148 Basics of Electrical E 3.9.1 Passive Network A network containing cireuit elements without energy sources, then the network is called the passive efemend and the elements of the network are passive elements. Elements, which cannot generate energY but can dissipate ar store energy are Known as passive elements, for example, resistor, inductor, eapaciter, 3.9.2 Active Network A network is said to be active if it contains energy sources together with other passive elements. An active clement generally supplies electrical enemy to the network, for example, battery, generator. 3.9.3 Lumped Network Physically separate resistors, capacitors and inductors are called tumped parameters. These elements will be connected together bY wires (leads) having practically zero resistance, then the network with number of lumped elements and a set of connecting leads, is called a limped-parameter network. In such circuit KVL and KCL holds good. 3.9.4 Distributed Network A network is said to be distributed if the network elements (&, 1, C) cannot be ically scparated und individually isolated as separate elements Examples: Transmission lines, winding, of transformers and generators. If the th of transmission line increases or decreases, value of effective (R, LC) changes and cannot be separated physically 3.9.5. Bilatoral The elementsto be considered for electric networks are assumed to be bilateral when the voltage and current relations are same irrespective of direction of flow of current Ex- &, £, C, etc. However, for unilateral elements the voltage and current relatjon are different for two possible direction of flow of currents Example: diodes. 3.9.6 Time Invariant A s¥stem or network in whieh the parameters do not change with time is called a time-invariant system or timte-invasiont network. Thus, in a time-invariant system the relation between its response and excitation always remain the same, regardless at the time of application of the input. Time invariant systems are also called constant-parameter s¥stems, D.C. Network Analysis 149 A network is called sme varying if it contains one or more time-varying elements, 3.9.7 Network/Gireuit An elevirical network is en interconnection of active and passi such as resistance, inductance, capacitanee and sources elements, An electrical circuit is a network that has a closed loo giving a retum path for the current, A network is @ connection of we or more camponents and may not necessarily be a circuit 3.9.8 Circuit Parameters Constant of a cirewit are paramet between two variable (voltage, current), rs and characterizes bY the relationship A circuit element is ideal, when its voltage and current are related bY’ + Constant of proportionality or + A ditierential or integral relationship These relationships can be shown to be linear which means that an ideal circuit clement has lincar behaviour Most practical circuits ean be modelled by suitable intereonnection of these elements like resistance, capacitance, inductance and sources 3.9.9 Branch A line segment representing a circuit clement is called a b elements conneeted between two nodes constitute a branch, ch or number of 3.9.10 Nade End point of a branch is called a node or node is a junction when two or more than two branches mect. Fundamental node (junction) is ane where more than two branches meet (e.g, (4) and {1, 7, 8). 3.9.11 Path The movement through elements from ene node to another node without repeating any node is called a path. 3.9.12 Mesh and Loop A sel of branches forming a closed path is called a loop and a loop that does not contain any loop within it is called a mesh network having a 7 branches 6 = 7 Given network having a 6 nodes n= 6 {1, 7,8 are same node so treated as aone node} 150 Basics of Electrical E @ i), . “iy * |® — or (Mesh ar\, tee. » BV Cony cop ot mesh - d oO @ =) Above network having 2 2 meshes or 2 loops + Hoop (a be da)(which ot a mesh because this loop contains-an another loop (1) and ¢2)) 3.10 SOURCE TRANSFORMATIONS: Voltaze source having a series resistance may be transformed into a current (1) Source having a parallel resistance and vice versa. R Ve ay : > EO 3 Transtorm ‘Voltage sovrea having + Curent source (WiR,Jandaa a series ressisnce) parallel resistance R VeHR, Transform inia a Cumrertsourcenaving a > (Wollage source (IR, )haviog paratel resistance R, series fesistenee) sources cannot be connected in parallel unless the two sources have identical voltages and similarly the eurrent sources cannot be connected in series unless identical. The paralleling af voltage sources with non-similar voltage, resulis in heavy currents and damage equipment (2) Vol B.C. Network Analysis 154 WG as ve Wavy Mp oVy Ys 2 © -——o MeV, o ——o Dv LI. —_o (Aresistorin peralictwith avollage source marybe ignored arorritted entirely fem natch) © @, Oy ——> Disteyeh oO ——o Wt — Diet Urheberrechilich geschiitztes Material 152 Basics of Electrical E 3.11. VOLTAGE DIVISION RULE Consider n resistors that are connected in series. The voltage F, across any resistor &, is . z, v-in-|——_*& _lp in, lavas =z here, J” is total applied voltagi 3.12 CURRENT DIVISION RULE Consider er resistors that are resistor &, (i= 1,2...) is connected in parallel, Then, current f, through any a UR, +UR, +1 Fis the lolal cument (F= fy + dy #5 + oe * de D.C. Network Analysis 153 3.43 KIRCHHOFF'S LAWS There are two Kirchhoff's laws, which have an efficient method for calculating the currents in the branches of « network ef conductors. These laws are given below. 3.13.1 Law 1 (Point Law or Current Law) ‘The algebraic sum of the currents at any node or junetian in a network is zero Let fj, fy and d3 be the currents flowing towards the junstion and dy and Js be the currents flowing away from the junction as shown in Fig. 3.1 Fig. 3.4 If currents flowing towards the junction are assigned positive signs and those flowing away from the juniction at negative signs, then Ltht hbk 90 31) or, Le hth shat (3.2) Thus, the currents leaving a junction are equal te the currents entering the junction, 3.13.2 Law 2 (Mesh Law or Voltage Law) In any closed foap of network having active and passive clements thealgebraic sum of the potential drops af passive and active clements around the laop or a as use, E, $ oe 154 Basics af Electrical E mesh is zero, [f we consider that the drops in register in the direction of current is positive and take voltage source with sign (+ve er —ve} occur in the way, Let us travel from 4 to B to C to Bio d in clockwis IR) ~ B+ 1yRy+ By + By IyRy Ry = direction then. TR+1R,—1R, 1,8, =6 -6,- 5 3.14 MAXWELL’S MESH OR LOOP METHOD: In this method any network is div Jed into meshes and in each mesh a separate current is assumed to circulate. Th jon of all the mesh current can be token cither as clockwise or anti-clockwise, the solution becomes more Ystematic if the directions of all currents are assumed to be the same, Method for finding the mesh eurrents R Reo eR, I Dew @e, => Above network can be divided into 3 meshes => Let each mesh have a cusrents J), fy and fy RE eR t 4) B Mosh D.C. Network Analysis 155 For systematic solution take all the currents in one direction clockwise. Ist taking a Mesh ~ 1 and apply KVL, Branch & is common with mesh 3 and branch R, is carnmoa with mesh 2 By + IR, + (Uy 1 dRy +U) —L)Ry- Ey 8 TR, +R, + R)— LR, BR. We are going in the dircetion of f, Now take mesh 2 aad apply AY. in the direction af mesh currents it Ey fl) Mosh2 Apply AVL in the direction of fy Cn ORe + =I, hk, Ey= 0 pine decoioe of hy dragnet a Apply KVL in the direction of J (fy FOR + (y= Ry +R AR, — 1,R, + FOR, +R, + R= By ou(3) Solving (t) (2) and eq. (3) we can find the mesh currents. F.=0 156 Basics of Electrical E 3.15 METHOD FOR WRITING THE MESH EQUATION IN MATRIX. FORM (i) Let chi be divided into a.m number of meshes. (ii Namber of mesh currents (fj, /5, ~/yf,) is sme as the number of mesh. Assume all mesh currents are in same direction. Gil} Let Ey, Ey. E,, are the algebraic sum voltage sources of each mesh. Ry h E h E; h Ey Ry Ry 4a} LE, ERlvcon = mesh alae atm MEO EM VETO ge Vallge ome => Ifonly voltage sources and resistors are present, then All the resistances through which the loop current J, (current of j"" loop) flows in the |" loop (j= 1, 2, 3... m).are summed and the sum is denoted DY Ry Apps Ros, Ry Byg)- Then the coefficient of fis R, is taken with positive sign. R,;is called self resistance of loop or mesh (v) Allthe resistances through which the loop current f, (current af; loop) and laop current /, (current of &'" loop) flows are summed up and denoted bY Rig (Rj3. Bix Rigs Hay, Ags ~ Ry...) then the sign of My is negative. Ag isealled mutual resistances (common resistance) beween the /" and k" meshes (vi) Lot £7 be the effective voltage in the ¢" loop through which the loop current j; flaws. The sign of the term £; is positive ifthe direction of E. is same ay that of J, (ot current ff enters to a —ve terminal of B). (vii) For networks having only passive elements and without any dependent source, the resistance matrix becomes symmetric (Ry = Ry;) (vill) Number of mesh e 4 umber of branches ms are m= bots n> number of nodes 5 number of separate parts D.C. Network Analysis 457 Example: Write the mesh equation in matrix form for the given Fig, Total no. of branches b= 10 Nodes 0 Number of mesh equation a — 10-841 =3 Resistance matrix is form of 3x 3. Rf] [Fr Ry {fds | ~ | BS Ry Ld ES Ruy Ry, = mutual resistance having currents 4, and f, ==(R)= Ry Ry, = Ra, =~ (R,) = common resistance between mesh (1) and mesh (3) Ry = Ry + Ry + Ry Ry = Ry + Ry + Ry * Ry (otal resistance of loop — 3 having eurrent J.) Ry = Ry = AR) E| = effective voltage of loop — 1 =-£, E} effective voltage of loap — 2 EY = effective voltage of loop ~ 3 ‘Ono source 158 Basics of Electrical E So matrix is (B+ Ry 4 Ry Ry Ry 4 (Rt RAR) Ry I =k, (+848, +8)| |, Ans. 3.16 NODAL ANALYSIS Mesh analysis is used to find out unknown currents in the mesh, Nod] analysis is used to find the unknown voltage at different nodes. Steps for Nodal analysis (i) Sefect a datum node or reference node and assume its potential to be (ii) Now select different suitable nodes and assign them a voltage with respect to a datum node. Gil) ATT branch curr ed ourvard from a node at which a nodal analysis have to applied and assume that node should be st higher potential Apply KCL at each node of unknown voltage and write down the equation in terms of node pair voltage and cixcuit parameter. => Let us considera general branch having a current /, autwards from node 4 Vand V, are node voltage 1 is as R, R, R. Vy an . 4 5 i Fag = V4 — Py = ARE FER: + By bE Ry Ey-(E,)—V, _ __ Voltage between node fand B B+ RR, Total resistance between nodes 4, = I; isoutward from node 4. Node A will be at higher potential, so we will subtract all the voltages from 1”). D.C. Network Analysis 159 Now consider a vineuit ve Be % ee ° oRy Ry Wc j j On, @e, ° wri (i} First, select suitable nodes, We will select « node where more than two branches are incident. Select one nede as a daturn node: (ii) Assign a node voltage to all the selected nodes. Lethe node voltages be F,, Vy with respeet to datum node f= 0 volt {iii} Now assign the direction of current at the node at which we have to apply nodal analy: Nodal anatysis at node 1 {a) We will assume that all the branch currents are answard from this, particular node, (b} We will assume that this particwkar node at which we have to apply nadal analysis should be at higher potential (c) Apply KCL at node | hehe 160 Basics of Electrical E Nodal analysis at node 2 {a) We will assume that all the branch currents are outward at this particular nade, (b) This particular node should be at bi + potential (c) 4 branches are connected to node 2 Re L th zr, Apply KCL at node 2 he Hehehe bao he 3.17 CHOICE OF METHOD MESH OR NODAL (1) The number of voltage variables equals the number of fundamental nodes ar junetian minus one. Nodal equation ={n ~ Iyandevery voltags source connected to reference node reduces the number of unknown bY one. (2) The number of current variables equals the number of meshes. Every current source ina mesh reduces the number of unknowns bY onc Number of mesh equation = 6 — 2 +1 (3} Mesh analysis only applies to planar using only two dimensions) (4) Nodal analysis can be applied ro both planar and son-planat eirenits. The method with the least unknown to solve is selected. For cireuits that are nonplanar and cannot be redrawn ina planas form, then nodal analysis is only the ehoic uit {citeuit that can be drawn D.C. Network Analysis 161 3.18 WHEAT STONE BRIDGE Wheat stone bri ge consists of four ratio arms 4B, BC, CD and Da and two conjugate arms 4C and 2D as shown in figure. Conjugate arms are those arms for which the condition remains unchanged by interchanging their positions When the bridge is balanced, no current flows through the galvanometer and hence # and ) are at the same potential. Vea — Va Va — Va Py ~ Vpand Vy Mot iP =i8 (1) Vac = Kae Q=|R nt From (1) and (2) S| g eR n unknown resistance § can Tf the valuc of R and ratio P/Q are known, be calculated 3.19 DELTA-STAR TRANSFORMATION A transformation in which delta-coanected resistance can be replaced b} equivalent star, so that the resistanee measured between any’ two terminals is, unchanged, is known as deliaestar transformation. When 4 circuit (a) opened, the resistance between the terminals B and C of 162 Basics of Electrical E . >e . a + + 02 s ad 5 : * (ay b) 6 And the resistance between terminals B and C af circuit (b) rir Tf the circuits (7) and (5) of figure are identical, then the resistance between any pairof lines will be the same when the thied line is opened. Hence. nalts ta When dis opened, ryt ry = Nt wD Fy thy ty When Bis opened, ry ry = When Cis opened, r+ ry = Adding eqns. (1), @)and (3) together and dividing both the sides by 2. we obtain, ray thi Her rte (4 OE a th th a Subtracting eqns. (1), (2) and (3) respectively fram egn. (4), we (8) (6) and, (7) D.C. Network Analysis 3.20 STAR-DELTA TRANSFORMATION From eqns. (5), (6) and (7), we obtain ry lt trytral=rary Pala Pst Pal ttt) slr i Dividing eqn, (10) by eqn. (8) Fry = rar 27H on on, on ry Similarly, 163 (8) (9) (10) of ID 12) 3 aa) (13) 164 Basics of Electrical E 3.21. THEVNIN'S THEOREM Theynin’s theorem states that any two terminal linear network containing energy sources (voltage or current} and impedances or resistances can be replaced with an equivalent consisting af a wallage source (Fy) and a series resistance or impedance (Ry, or Zy) evnneeted to load. Anynetwow | | ow naving neat torments ® nergy sources: 1 Load (Deperdentor "ndependert) *y «) 7-—*_| Rat By Thevnin’s thearem is especially useful in analyzing pawer systems and other circuits where foad is subject ta change and recalculation of the circuit is necessary with cach trial value of load resistance, to determine voltage across it and current through it So, Thevnin’s theorem is away to reduce a nctwork toan equivalent circui camposed of a single voltage source, series resistance and a series load, 3.22 STEPS TO FOLLOW FOR THE THEVNIN’S THEOREM (1) Find the Thevain saurce voltage (F,) bY removing, the load resistor from the original circuit and calculate the voltage across the open conncetion point where the load resistor used to be. Lingor Linear active R > acwe otvark ater etwork > removing . load (2) Find the Thevnin resistance (R,,) by removing all power sources in the original circuit (voltage sources shorted and current sources apen) and then calculate total resistance between the open connection points D.C. Network Analysis 165 emer | | kee nase aR > | ie R= Re woes remove | somone I (eos como fovaurces (3) Draw the Thevnin equivalent circuit, with the My in series with Ry. The load resistor reattaches between the {vo open points of the equiva circuit ent 3.23 NORTON’S THEOREM Norton's theorem states that any two terminal linear networks containing: energy sources and resistances or impedances con be replaced by an equivalent circuit having a current source (Uy) and parallel resistance or impedance connected to a load Nawork (A) containing inear elomants and energy sources: (dapendentor indopendtent) © + So, Norton's theorem isa way to reduce a network to an equivalent circuit composed of a single current source, parallel resistance (} and pralle! load 3.24 STEPS TO FOLLOW FOR NORTON’S THEOREM. (1) Find the Norton source current (/,) bY removing the load resistor from the original circuit and starting the open terminal. This short circuit urrent 18 hye Native (AD 8 alter amoving ° ccantaining theload near alamonte % and energy sources Losd > rh (deponcern or Pdagendent) 5 meena 4 166 Basics of Electrical E (2) Find the Norton resistance (Ry = Ry) by removing all energy sources {independent voltage sources shorted and independent current source ‘opea) and calculate total resistance between the open connection point after removing load. etwark (A) containing . llnger stements Race, (abi tre ane terminal butnesdependent - ‘or rerrosing load) . (3) Draw the Norton's equivatenteireuit with the Norton’s current source in parallel with the Norion’s resistance. The load resistor reattaches between the wo open points of equivalcat circuit, 3.25 MAXIMUM POWER TRANSFER THEOREM This theorem states that the maximum amount of power will be dissipated by foad resistance when that load resistance is equal to Th of the Network supplying the power. If the load resistance is lower or higher than the Ry or Ry, its dissipated power will be less than maximum, nin/Norton resistance Application Practical applications of this might include sterco amplifier design (seeking (0 maximum power delivered to speakers) or electric vehicle design (seeking to maximum power delivered to drive motor), In stereo system design speaker “impedance” is matched to amplifi output Proof: Fora DC Network “impedance” for maximum sound power Pu bok D.C. Network Analysis 167 Power delivered ty load R, is P, Patek FER, (By + RP To find the value of R, that absorbs a maximum power from the given practical source, we differentiate with respect to Ry. APL (Ry +R) NG ~FER VR, + RY aR, (Ry, + RY Equate derivative te zero. = Ry (Ro, + R= (Ry RL = => Anctwork delivers the maximum power to a load resistance R, when R, is equal to the Thevnin equivalent (or source) resistance of the network, 3.26 MAXIMUM POWER TRANSFER THEOREM FOR AC NETWORK* ‘The power transferred from an active network wo a load depends on load, Notwork (RL) a ‘Sources at [lee My, i (i) Load is a pure resistance. (ii) Load resistance and load reactance are independently variable. (iii) Load has a variable impedance bul a constant power factor ‘Nr OPT E7980) set 168 Basics of Electrical E We have to find the value of 2, which reecives or absorbs maximum power from the network © By RD Sat ED) ¥, (Ry + RY +X at MY Average power ransferred 10 load is P, - oR (Ra RY OX LY R For maximem power wansfer theercm &; and kt, will be varied independently. a, a, pu 9 and <0 OR, ak, (i) Let us consider varying X; and keeping R, constant dP, 2 0 ax, O-2V PR, (ky + Ky) [a Ry)? +X XP [X= Xa : ah, Gi} Now, Ay and aR, =o P, Foy + Nw (By +R 2, at, 4.4 VAR, (Ry * RY] = 0 ae, an, Pehl m D.C. Network Analysis 169 TR, Ry Ry Ry Ruy So, for absorbing maximum power bY toad Z,, 2, must be equal 1 = Conjugate of Z, Hence, maximum power is transferred from an AC network to load if the load impedance is cqual to the complex conjugate of Thevnin equivalent impedance. Z=2y Case 2: Consider &, and X, are net independently varied but only magnitude (Z,) of load is varied for constant power factor (= o= Z| : 4 Hig Ry © (RARE +O > XD 2,005 9 and st, = Z;, sin & pz) dz, ar | rere lo. + Z, cosg) cose + 2X, (A, cos § +4, sin @) + 2] ft Zh cot 6+ DR BeOS ++ Zi sin gs Wes ae 2A Asin +227 =z Wal = 1721 So, when magnitude of load impedance is varied but the power factor (cos 6) remains constant maximum power is transferred when |Z, | ~ |Z, 2, sin pb sing] 170 Basics af Electrical E In transformer to achieve maximum power transfer, where Z, can be adjusted but 6 cannot be. In AC network when |2;| = R, for unity pawer factor. Load absorbs maximum power when 3.27 SUPERPOSITION THEOREM This theorem states that in a linear network, containing more than one independent energy source, then the complete response (vollage ar current) in any branch of network is equal to the sum of the response due to cach independent source acting one at a time with all other ideal independent sources are made inactive (short the voltage sousce and open the current source), 3.28 STEPS FOR ANALYSING A CIRCUIT (1) Take an¥ one independent source in the circuit. (2) Make all other independent sources inactive. (3) Dependent sources will not be disturbed. (4) Determine the magnitude and direction of response (voltage or current) in the desited branch by a single source selected. (5) Now take anather independent source and calculate the response in a desired branch using step 1 10 4. (6) Add all the component of responses in the di addition is to be done for DC networks and phasor addition for AC network, tations (1) Superposition theorem annat he applied for caleulation af power. (2) Superposition is the combined properties of additivity and homogeneity of linear network. [f any one property is violated then principle of superposition is not valid. (3) Applicable only for linear networks: (4) Properties of additivity and homogeneity are fundamental to superposition principle, (5) There should be more than one source, Additivity IFX, is the excitation (cause) and F; isthe response of given network, related by = cx, D.C. Network Analysis 471 Similarly for an excitation X; response ¥, = CX, Lf the network is linear, then for excitation LX, Oty) (2) the response will be + Y= CON +X) If the excitation and response are related by the linear equation CX, +d), The network js not linear as for double the excitation response will not double. Homogeneity: Ifall the sources are multiplied by a constant, the response is multiplied by the same constan ut ¥=CX Let excitation be multiplied by m become mx, Then response ¥ = Clmxyy= oY, SOLVED PROBLEMS Points to be remembered (1) Find 14, = Fy — y= Potential difference berween 4 and B. RY rR, Ae eh nike “5 (1) For finding the potential difference between twe points, start from one point and reach to smother point by any path, (2) On going from 4 to B, add all she voltage drop (IR) and source voltages (5 ¥2 (3) Ifwe are going in the direction of current take drop positive otherwise negative, 08 MapsIR}—» intradrectonoteunen tom ALE ase In the osposite direction of eurent i from At Ac ©B May" Vy) ® On going fom Ato b+ Sign ccourseon theway ¥, Ao 77 a8 ag #14) — On going fens M10 Be sion occurs conway 172 Basics of Electrical Vag = By ~ Vg = aR, + (BP) * UR, AR, = (V2) * IRS yp = IR, Vy + UR, + ARs + Vy + Hy (4) Ifwe know the node voltage V, atid My of My, We can find the branch current J. J is the current going owt from node (4). F _ Total potential difference between A and B R ‘Total resistance for path 4 to B 6) Vor finding ¥” start from node 4 and subtract all the voltages from V, with a (6) R wee Mao awe Fa 1a ae RY Varo Aeris “a a 1s Yow Row Ye ae * ep pe MMMM aera 3 ie Example 1: How many independent loop equations are there in the given network shown? D.C. Network Analysis 173 Solution: Total number of independent loop equations are m mebantl Since, there are [4 branches and 8 nodes, branch (il #8 a part of between two nodes). m= 14-8 1= 7 equation Example Solve the given circuit by mesh analysis and determine the current drawn from voltage source F, 42 Solution: Total number of nades = 4 Total branches b = 6 Total number af mesh equations =~ 4+ | Total number of node equations = n — 1 = 4 a) 174 Basics of Electrical E In mesh 3: h=-2Amp 2) mesh 2: Mh-h)th+ Fh hyn 0 (3) -24,+ (4) Solving for (1) (2)-and (4) we get = 2.Amp, y= b Amp. fy Curent from source V5 is ,=2Amp Ans. 2 Amp ample (i) Voltage source having a resistance in s parallel resistance. Using source transformations, simplify the given network into a source having @ 1a@) g2a 378 Solutio t ‘Corwen sittno vollage sources (toa ourent source, 3a) D.C. Network Analysis 175 Example After converting the current source find / im 20 resistor by using mesh analy sis an Ag s120 LF Ww 2a) oi 2120 20 7 av a Solution: 10 ia me Me 2a => 20 pen " uy Apply mesh analysis in mesh 1 A(U2+ 1 fy 2 mol LY Mesh 2 Ugo A). 1 4 Le y= = Dey 22) From equations (1) and (2) 176 Basics of Electrical E Rede Amp. Faample ce athe bridge circuit shown in figure. 2 © Solut . 20 Covering the delta to star, Te} 1 12 4 2 D.C. Network Analysis 477 Example 6: Find the current through resistor 2; = 40, by using (6) Nodal analy sis. (7) Mesh analysis ae (8) Superposition theorem (9) Thevnin theorem @rev (10) Norton theorem t Solution: (6) Using Nadal analy sis: Af s aot fan rove) sa Sow {a} First select a datum or ground node. (b) Now select a valtage of node (Il) is ¥ {c) Number of branches canneet from node (1) is 4 (ly Assume all the currents from node are outward, wove) (©) Total sum of current at node is zero Aththth-0 Samp 178 Basics of Electrical E ‘Vallage at node (I)is 7 Current through 8, is J Amp (7) Using mesh analysis: (i) Assume a loop or mesh current in each loop) Gi) Apply KVL in each loop 20 roves Loop @) = 10+ 21, +2, -1,)=0 Applying mesh analysis loop — 1 10+2 +2, -h)=0 (1) D.C. Network Analysis 179 Applying mesh analysis in loop — 2h — A) + AU) = 0 We can’t apply mesh analysis in loop (3) because there is a current source. But /,=—5 Amp Alice solving (1) and (2) we get i . he -3 Amp Current through R; is > (8) Using superpositis wv) samp Case =1: Taking a voltage source of 10 V and deactivate other sources here current source, So open the current source, and find Jy. 22 ane wv) 180 Basics of Electrical E Using nodal analysis > 5 V=4 Volt hh tat 1 Amp J down Case — 2: Taking another source (current source) and deactivate 1 sources here it is @ voltage source. So, short the volt its internal impedance, other ¢ source or replaced by 22 p< sc 1 Amp 4 downward Now according to superposition principle net current through R; is #. [=hth =l4l I=2Amp (9) Using Thevnin’s thearem: RytR, So, find # we have to find Py and R, D.C. Network Analysis 181 a ‘ 4 2R, + 2a5 j ' L vee eR, weve Ou B “Trewin equivalent Gi) For finding F, open the terminal across which we have to find Sar have 10 draw thevnin equivalent, So Vy is nothing, it is a open terminal voltage. So open the AB and find yg whieh is Fy. 20 y A qd Now, Fg can be found by both mesh or node? Samp Ci) by mesh b= and, 1024) +2) -J,)=0 4f-10+ 1020 [7,=0 Amp] Fy= thm) = 40+ $) = 10 Vote =10 Volt 50, Gi) by nodal For finding Ry: () Open the terminal across which we have to find the thevnin equivalent, Gii) Now deactivate the independent sources (a) Current source will be open. (b) Voliage source will be short or replaced by its internal resistance, 182 Basics of Electrical E ) Now find the equivalent resistance between open terminal 4B. aa A bot yg = Ry = 22 | ang a= Ry =2| se . 2x2 ' Ru- 55 12 . 242 a Sof= 9 W,=7) Volt eT ton Va 2—vy—9 Ve. “ TA 3 = Pag 10h ° Apply node analysis at node B: 3 branches are al node 8. Assume all the currents at node B is outwards, L+h+h=0 ta od ,We-D ,4 4 (1) Nodal analysis at node C: 3ebranches connect at nede C A+hth=0 186 Basics of Electrical E Fav, +11. =21 (2) Mes Solving (1) and (2) e=2Volt] ve ve =-2 Volt Ans. (13) Using superposition theorem: In superposition theorem we will take one source at a time and rest of the sources will be deactivated, tS aa va) a TAD sad, 208 Case 1: Take a voltage source and deactivate the current source. apply nodal analysis at node C WoaT ee, D.C. Network Analysis 187 Iw ing gan 1V@ bom, a ey Case-2: Taking (74) current source and eesetivate voltage source, ‘oven oe pa wy 28 vy TAD) ane at node-4 (Ly at nade ~B 198 Basics of Electrical E 14) Using Thevnin theorem: $ Rye re > . 8 Thevnin equivalent Fs -Ry and P= For finding Vy: (i) Open the terminal AB, then find I"y,, that #44 i8 Vy (open ckt voltage) Now Wy ¢an be found by Mesh or Nodal analysis. (a) First by Mesh Mesh 2 is open so A-f=7 Amp So 1-7 Amp D.C. Network Analysis Inmesh 3 9-193, (b) By nodal > at nade-C > at node — 4 189 2,0 Gh B 0 = |fy= = Amp] Vag = Start from d reach to 8 by any path and add all the Srops of that path with sign Vgg--2igt+7 7 2241, dx] vv" 6 i 14 - Vy = Volt) =F Ab ot a o Kr a Volt 190 Basics of Electrical E For fi Short the voltage source, open the current source, then find the equivalent tesistance between the open terminal 20 22a a2 } Bag oa, a Ryw= B+ OI2 | 4x2 6 nas waa) 4 D.C. Network Analysis (15) Using Norton's thearern: > Wh) me ik For finding fy, short the termins current. we yf ae 20 we ne hay 7mL 20 I / tye dy Lyeoly=7 Apply combined mesh analysis in loop ~ | and 2, <1 +U,—)+ My = h)=0 40, +30y Apply mesh anal (fp- f+ ML + I= 0 lad, — Bly =O) 491 At) (2) 192 Basics of Electrical E Put 1, = Fy *7 in (2) and (3) [eT -4h* y= 7 in) from —3 6ly-Iy-T-My=0 Ry iv same as Ry vt tema By San ” (Norton's equivalent) 4 1 eo 7-3 S47 Ans. D.C. Network Analysis 193 PART-B Example 16: Find / using— (16) Norton's theorems ; 5 (17) Thevnin’s theorem (18) Mesh analysis Fen (19) Nodal analy sis mae) (20) Superpasition Prov 1 Solution: 8 (16) (i) Norton’s theorem: a - 7 én, san ® Nonan’s equivalent Ry Ry #2 For finding far for drawing Norton's equivalent ekt. We have to find (i) Ry For finding 2, (a) Deactivate all the sources, short the voltage source and open the curre souree. (b) Now find the equivalent resistance between the open terminals (48) across which we have (@ draw the Nortan’s eq 194 Basics of Electrical E ie Ise 0F I Jy is nothing but a short CRT current. So, short the terminals (across which the short CKT current have to find) and find current fe using Mesh or Nodal analysis. Noda _ 2 oY A Sea San he tow av 8 a 1. 20Amp, ls" Te + Alla shored hence Vp = 0 Volt nooo? £0 apply nol at node {7h Bl, dl, + a0 +1020 vito vv, “ete 78 [77 47, = 110] a) but Vv; =0 In loop-3 TAV, = 750 aly aly = 40 i, = 10] (2) Pute into) 17h, = 170 = Aly D.C. Network Analysis 195 Ti, = 280, b= bse [, = 2809] 280 3 375 amp Now same has to be done by Thevnin theorem. (17) Thevnin theo (Thevnin equivatent) (a) For finding Ray Ry is found sameas Ry, (by For finding Vy, Vg is nothing it is the open CKT voltae across the terminal across which You have to find Thevnin equivalent CKT, So open the terminal AB and then find V’,. ~ Vyg© Fy BY Mesh or Nodal analysis 5a A Zen 104 3170 @ rv 196 Basics af Electrical Using Nodal $ bt s gsa 2oate i nat Quy Dav Quv Dav 8 E 8 Vga forfining Vjg you have foreach | Vag =opon CKT vottage hance Alo By any path f= al + €O] for fining | 5 apply mesh analysis in mesh 2 aE © 1714 40-10 = 1600 —_—_= Yaw | y-40 j= 110 es apace 7 WW, @) 110 eVye dx 18 440 - v Put y, = = 200 7 v, = 1M ~ 65. Vos - L207 Ans. Same problem is now done by Mesh and Nodal analysis DLC. Network Analysis (18) Mosh 5a iis 41, * “ ROM 2m [== 7.4 = 20 Amp Apply Loop analysis in (2) Bllp~ 1) # Ble + AGL = Apr # 10= 0 Th; —4ls 81, + 50= 0. 1604 50=0 17,4 = 10 Mesth analysis ia Loop (3) a4l, # 16h, = 40 nh +48, = 10 from (A) and (B) f= 4375 21 Ans. (20) Same prablem is by Superpas 197 (19) Nodal $0 a San faa Zn0 200) Hv 40 yt a At node-1 vey 3 3 iV, = BY, = 750] Putin (1) (2w, - 400) ae jon Theorem, pa A ao ait 128 B There are three sources hence, current / is the sum of current by individual sources by di source), 1 rnp A ctivating other sources (short voltage source and open current ho vn * Hap vane 198 Basics of Electrical E Case t: Taking 20 Amp source and short other voltage source. 52) A Fea fan F180 20 Amp (sc G) se. Th anp 8 4 by current divider rule = «dT (4419 sg and 8+R and. R=54 (412-82 8 i= —— x20= 10 Am Bes P Ato 5 amp b= 2.5 A downward (4 16 B) 16. Case 2: Taking 10 V battery source say 4 na San fan ren oc! rv [se tu 8 r ove e av 5 Apply nodal at node 1 rev oe +742 50 B42 D.C. Network Analysis 199 48 V+ 1561+ 520 = 480 480 v= w 356 "OH i, - 48 dow atx Amp 4410 ¢>~-186.A (downwards) Case 3: Taking 40 V voltage soures the other voltage source. wd open the current source and short sa at Sen Eve 2)40¥ rie => Fi r 2 Ig 0° 256« E a3 266 1, = 2.03 A (downwards) Be 2 fi

Das könnte Ihnen auch gefallen